Categories
Bryn Mawr Chicago Economists Gender Home Economics Illinois Radcliffe

Bryn Mawr. Economics Ph.D. Alumna. Lorinda Jane Perry, 1913.

 

This new entry in the series “Meet an economics Ph.D. alumna/us” features the 1913 Bryn Mawr Ph.D., Lorinda Jane Perry. Details about the last 25 years of her life are relatively scarce compared to the events leading up to her last academic position as an associate professor at Hunter College in New York City, i.e. up through the first half of the 1920s. She apparently left economics to go to the Law School at the University of Chicago and as of the 1940 Census was sharing a home in Chicago with four likewise single siblings (a former member of the Illinois Legislature, an attorney, a urologist in private practice and a medical doctor working in the Health Department). 

___________________

Lorinda Jane Perry
Timeline

1884. Born December 23rd in Melvin, Illinois.

1900-1904. Illinois State Normal University.

From the Index, 1904 Yearbook of I.S.N.U.

While in high school I burned with a desire to know all of the latest slang. But that fire has been quenched. Now I can’t bear such expressions as “Oh! Deah,” or “By Jinks” and others. Now I see the wrong and wish to form a society for the “Purification of the American Girl’s Language.” I have not outlined my course of action, but hope some day to sing with the poet:

“Hail to the graduating girl, who is sweeter far than some,
Who when she talks, speaks no slang and chews no chewing gum.”

Between 1904 and 1906. Lorinda Perry taught in country schools near Melvin and Monmouth, Ill.

1906-1909. A.B. in Economics and History at the University of Illinois.

1909-1910. A.M. University of Illinois. The History of the Lake Shipping Trade of Chicago. Simon Litman, thesis supervisor.

1910-11. Women’s Educational and Industrial Union Fellowship at Radcliffe.

A fellowship of $500.00 established and maintained by the Massachusetts Federation of Women’s Clubs and the Women’s Educational and Industrial Union, 1905-1909, has been continued by the Women’s Educational and Industrial Union for the year 1910-11. This fellowship is offered to a graduate student who has been recommended by the Professors of Economics in Radcliffe College. The holder of the fellowship must devote one year to research under the Department of Research of the Women’s Educational and Industrial Union with a stipend of $500, and one year to graduate courses at Radcliffe College with the usual tuition fees as stated in the Radcliffe College catalogue; or she may devote one-half time to research work at the Union and one-half time to graduate courses at the College for two years, with a stipend of $300 per year. Applications for the year 1911-12 should be made before May 1, 1911, through the Dean of Radcliffe College.
The fellowship was awarded in 1905-07 to Caroline Manning (Carleton College) A.B. 1898, (Radcliffe) A.M. 1907; in 1907-08 to Grace Faulkner Ward (Smith) A.B. 1900; in 1908-10 to Edith Gertrude Reeves (University of South Dakota) A.B. 1906, (Radcliffe) A. B. 1907, A.M. 1910; in 1910-11 to Lorinda Perry (University of Illinois) A.B. 1909, A.M. 1910.
Source: Annual Reports of the President and Treasurer of Radcliffe College 1909-10, p. 66.

1911-13. Graduate Student at Bryn Mawr College. Fellow in the Department of Research, Women’s Educational and Industrial Union.

1913. Ph.D. Bryn Mawr. Millinery as a Trade for Women. New York: Longmans, Green, and Company. Susan Myra Kingsbury and Marion Parris Smith, dissertation supervisors.

[From the Preface, written by Susan M. Kingsbury, pp. viii-iv]

“In the fall of 1910, Miss Lorinda Perry, a graduate of the University of Illinois, 1909, securing a Master’s degree in 1910, and Miss Elizabeth Riedell, a graduate of Vassar College, 1904, were awarded Fellowships in the Department of Research of the Women’s Educational and Industrial Union and selected for investigation the subject of Millinery as a Trade for Women. During the year employers and employees were interviewed, and the results secured from the former were analyzed and interpreted by Miss Perry, from the latter by Miss Riedell.

In the years 1911 to 1913, Miss Perry held a Fellowship at Bryn Mawr College and under the direction of Dr. Marion Parris Smith, Associate Professor of Economics, continued the study of the millinery trade in Philadelphia. Miss Perry’s discussion of the trade in the two cities was accepted by Bryn Mawr College in partial fulfilment for the degree of Doctor of Philosophy in May, 1913. In Philadelphia the field work was conducted by the Consumers’ League and at their expense under Miss Perry’s direct supervision. Fortunately the information on the trade in Boston was brought up to date by the courtesy of a number of Boston employers who permitted their entire pay rolls to be copied from their books by the secretaries of our Research Department. Tabulations of this data and retabulations of the earlier Boston material by our secretaries enabled Miss Perry to unify the two studies and to revise most of her earlier work and that prepared by Miss Riedell. Those sections dealing with the effect of seasons on Boston employees and on Boston workers in the trade as secured from personal interviews are therefore the combined work of the two students.

The method of attack, the range of inquiry and the extent of returns in the investigation are all presented in the introductory chapter. As this was one of the first studies of the type by the department and indeed in the country, the schedules were far from perfect resulting in an incompleteness which in later studies of the series has been avoided. It is to be regretted that the opportunity to use pay rolls came only within the last year so that detailed information as to wages was not obtained from the workers who were visited in their homes, as was done in the study of The Boot and Shoe Industry in Massachusetts as a Vocation for Women. It is also unfortunate that pay rolls could not be secured in Philadelphia.

Prepared for the purpose of affording students training in social investigation, the study must lack in finish of presentation and completeness of interpretation; but the work has been carefully supervised and supplemented by every means available to the Research Department. In order that the survey may serve as large a group as possible, the material is often presented in much greater detail and the tables arranged with much smaller class intervals than might at first appear necessary or desirable, although discussions in the text often deal with larger groupings. Indeed in many tables the facts are presented for each case, especially where subclassification has made the number considered too small for generalization. We hope that agencies interested in a study of minimum wage laws, in other regulation of working conditions by legislation, in vocational guidance and placement, in industrial education, and especially, in awakening the public conscience may each find here data which can be rearranged or grouped so as to form a basis upon which to act.”

1914-1916. Head of Department of Political and Social Sciences at Rockford College

1916. Dissertation published The Millinery in Boston and Philadelphia: A Study of Women in Industry. Binghamton,New York: Vail-Ballou.

1916-1920. Associate in Department of Household Science. University of Illinois.

DR . PERRY TO GIVE COURSE IN HOUSEHOLD ACCOUNTING

Dr . Lorinda Perry, associate in home economics, will have charge of a class in household accounting to be given under the auspices of the Home Improvement association of Champaign . The course will be open to members of the association only, but membership in the organization is open to any who wish to join. The object of the course is to teach the women how to place their homes on a business basis.

SourceDaily Illini, March 8, 1919, p. 5.

1917-1918. “Some Recent Magazine Articles on the Standard of Living,” Journal of Home Economics. Vol. 9 (December 1917), pp. 550-558. Concluding Part. Vol. 10 (January 1918), pp. 9-17.

1919. Taught in Chicago according to report in the Daily Illini, Nov. 22, 1919, p. 8.

1920. Appointed Associate Professor of Economics at Hunter College, New York City.

Ca. 1928. J.D. University of Chicago.

1926-27 Registration of Second Year Student, Lorinda Perry, Resident Autumn, Winter, Spring Quarters.
Source: University of Chicago, The Law School, 1927-28. In Announcements Vol. XXVII, no. 22 (May 10, 1927). p. 20.

1931. [Miss Lorinda Perry of Chicago] while in Melvin during the Thanksgiving season, learned that she had been successful in passing the state bar examination”. The Paxton Record (Illinois), Dec. 3, 1931, p. 10.

1940. U.S. census. Living with brothers and sisters, in Chicago Ward 5, University Ave. No occupation listed either for her or her older sister Josephine (who had twice been elected to the Legislature of Illinois from the Fifth district from 1930 to 1934).

1951. Died August 30th in Chicago, Illinois. Last residing at 6221 University Ave., Chicago.

 

Principal Source: Obituary in The Paxton Record (Illinois), September 6, 1951, p. 1.

Image Source: from the Holton/Kinney/Foster/Watson family tree posted at ancestry.com.

 

 

Categories
Chicago Cornell Economists Germany Harvard

Chicago. Economics Ph.D. alumnus, later Cornell professor, Newman Arnold Tolles, 1932

 

Tracking the careers of Ph.D. trained economists at Economics in the Rear-view Mirror has not been limited to the handful of tournament winning, prize economists of past times or even the prominent gatekeepers of orthodoxy. Our series “Meet an economics Ph.D. alumna/us” includes both those who have moved and shaken their local academic communities without leaving much of a footprint in the sands of the history of economics and those who have constituted the vast majority of economists who have survived the demands of the graduate economics programs of their times and then modestly contributed to the pool of our collective economic knowledge during the course of their professional careers.

Today’s economics Ph.D. alumnus, Newman Arnold Tolles (University of Chicago, 1932), achieved considerable professional success during his lifetime, though he is unlikely to ever be found in the syllabi of present and future histories of economics. Tolles is however worthy of nomination as one of a myriad poster-children representing mid-20th century U.S. economics. 

_______________________

Newman Arnold Tolles

Sept. 21, 1903. Born in New York City.

1923. B. Phil in economics, School of Commerce, University of Chicago.

1924. M.S.,University  of Chicago.

1925. Recent Literature on British Unemployment Insurance. Quarterly Journal of Economics. Vol. 39, No. 4 (Aug., 1925), pp. 651-662.

1926. A.M.,  Harvard.

1925-27. Study at the London School of Economics.

1929-35. Assistant Professor Mount Holyoke and part-time at Smith College in 1931-33.

1932, Autumn. Ph.D. U of Chicago (diss: Economic Aspects of Unemployment Insurance in Great Britain, 1911-31. Published Chicago: University of Chicago libraries, 1935).

1935-1945. Government service (1935-38 as economist with the Bureau of Labor Statistics, 1938-40 as assistant director and director of research in the US Dept of Labor’s new Wage-Hour Division., 1940-45 chief of the Working Conditions Branch at BLS).

(with Louis M. Solomon) Earnings in Eastern and Midwestern Airframe Plants, 1942 : Bulletin of the United States Bureau of Labor Statistics, No. 728.

(with Robert Julius Myers) Income From Wages and Salaries in the Postwar Period : Bulletin of the United States Bureau of Labor Statistics, No. 845.

Spendable Earnings of Factory Workers, 1941-43 : Bulletin of the United States Bureau of Labor Statistics, No. 769.

(with Louis M. Solomon) Wage Rates in the California Airframe Industry, 1941 : Bulletin of the United States Bureau of Labor Statistics, No. 704.

(with Theodor Winter Reedy) Wage Stabilization in California Airframe Industry, 1943 : Bulletin of the United States Bureau of Labor Statistics, No. 746.

1945-47. Professor and chairman of the graduate department of economics, American University.

1947. appointed Professor at Cornell’s newly-established New York State School of Industrial and Labor Relations to retirement July 1969.

1951. (with Earl Brooks and Richard F. Dean) Providing Facts and Figures for Collective Bargaining—The Controller’s Role. Ithaca: New York State School of Industrial and Labor Relations, Cornell University.

1952. (with Robert L. Raimon) Sources of Wage Information: Employer Associations. Ithaca: Cornell Studies in Industrial and Labor Relations, no. 3.

1953-54. Fulbright guest professorship in Munich and Kiel.

1957. New York State Department of Labor. Chairman of the minimum wage board for the cleaning and dyeing industry.

1959. American Minimum Wage Laws: Their Purposes and Results. Ithaca: New York State School of Industrial and Labor Relations at Cornell University, no. 95.

1960. The Purposes and Results of U.S. Minimum Wage Laws. Monthly Labor Review, Vol. 83, No. 3 (March 1960), pp. 238-242.

1961. (assisted by Betti C. Goldwasser) Labor Costs and International Trade (Washington, D.C.: Committee for a National Trade Policy).

1964. Origins of Modern Wage Theories (Englewood Cliffs, NJ: Prentice-Hall).

1965 study of salaries of professional economists for the American Economic Association [published AER Vol. 58, No. 5, Dec. 1968, Supplement, Part 2. Studies of the Structure of Economists’ Salaries and Income.]

1966. Weathering Layoffs in a Small Community: Case Studies of Displaced Pottery and Carpet-Mill Workers. Washington, D.C.: Bulletin of the Bureau of Labor Statistics, 1516.

1965-1969. Two terms as Ithaca city alderman as a Democrat.

1969. Lost race for mayor of Ithaca.

Two years after retirement part-time teaching at Cornell also teaching at State University College at Geneseo (economics department).

July 1971. Becomes emeritus professor at Cornell.

Apr. 10, 1973. Died from a heart attack while teaching his class at Geneseo State Teachers College.

Sources:

Cornell University Faculty Memorial Statement by Robert H. Ferguson, Vernon H. Jensen, Robert L. Aronson.

“Arnold Tolles Dead; Served County, City with ‘Compassion’”, The Ithaca Journal, April 11, 1973, p. 3.

Guide to the N. Arnold Tolles Papers. Kheel Center for Labor-Management Documentation and Archives, Cornell University Library

Image Source: From Tolles’ obituary printed in The Ithaca Journal, April 11, 1973, p. 3.

_______________________

All the World’s a Stage
Tolles @ Center Stage

Photograph of a scene from the 1932 faculty show. Verso reads: A Scene from Faculty Show, presented once every four years at Mount Holyoke College by members of the Administration and Faculty. They present ‘A Hard Struggle’ by Westland Marston, Esq., as a curtainraisser. Left to right: Miss Ruth Douglass of the department of Music, Leslie Burgeivin of the department of English Literature, Miss Dorothy Graves of the department of Art; N. Arnold Tolles of the department of economics; Miss Elizabeth Doane of the department of French; and Bernard Bloch of the department of English.

Source:  https://compass.fivecolleges.edu/object/mtholyoke:24371

 

 

 

 

Categories
Chicago Economists Exam Questions

Chicago. Preliminary Examinations in Economic Theory. Friedman, chair. 1952

 

Today’s post includes not only the questions for the economic theory preliminary examinations (Part I and Part II) from the summer quarter of 1952 at the University of Chicago, but also some interesting background material. From Milton Friedman’s papers at the Hoover Institution archives I have transcribed copies of the entire schedule of preliminary examinations for summer 1952 along with the correspondence between Friedman, Frank Knight and the departmental secretary. We can compare Friedman’s suggested questions with the questions that were actually used for the exam along with Friedman’s rankings of the anonymous examinations. Two sentences in Frank Knight’s letter to Friedman (after the grades had been compared among the graders and the veil of ignorance regarding the identities of the examinees was lifted) is definitely worth considering in light of current discussions about systemic elements of racism in the discipline of economics.

“I feel that these Negroes are in the same position as the Chinese students only more so in that they compete in a completely different market, and they are never really compared with our “full fledged” Ph.D. graduates. (Besides, between you and me, I have attended 4 or 5 Ph.D. exams this summer and thought very few of them ought to pass but they all did).”

I have gone on to track down the top eight examinees as ranked by Milton Friedman. Fun facts: Gary Becker won the bronze medal and Abba Lerner’s son, Lionel Lerner, placed fourth.

The summer 1951 theory preliminary exams were posted earlier.

_________________________

Schedule for the Preliminary Examinations
Summer 1952

July 15, 1952

To: Committee members of Preliminary examinations
From: J. Barker, Departmental Secretary
Re: Schedule and committees for Preliminary Examinations, Summer Quarter, 1952.

Date Examination Committee Registration
Tues., July 29 Economic Theory I M. Friedman, Chr.,
F. H. Knight
G. Tolley
26
Thurs., July 31 Economic Theory II (as above) 4
Tues., July 29 Government Finance P. Thomson, Chr.
H. Lewis
1
Thurs., July 31 Industrial Relations F. Harbison, Chr.
A. Rees
M. Reid
1
Tues., Aug. 5 Money, Banking & Monetary Policy L. Mints, Chr.
E. Hamilton
J. Marschak
21
Tues., Aug. 5 Statistics T. Koopmans, Chr.
W. Wallis
4
Thurs., Aug. 7 Agricultural Economics D. Johnson, Chr.
T. Schultz
P. Thomson
8
Thurs., Aug. 7 International Econoics L. Metzler, Chr.
C. Hildreth
H. Lewis
9

_________________________

Friedman to Knight and Tolley
Carbon copy

Orford, N.H.
July [19 or 20], 1952

F. H. Knight
G. Tolley

Dear Knight and Tolley:

I have just received word from Miss Barker that I am chairman of the Theory prelim committee for this summer, that you are the other members, and that the exams are to be in her hands by July 22.

I wish you could join me here for a session to get out the exams—and I am sure you do too if what we have been hearing about the weather in Chicago bears any resemblance to the truth.

Since you cannot, I enclose some suggested questions for both Part I and Part II. I wonder if the two of you could get together and combine these or such of them as you think worthy of retention with your own questions. Time does not permit of rechecking with me and I assure you I shall be more than satisfied with whatever decisions the two of you make.

As to the papers, have them sent to me at any stage that suits your own plans best, since mine are very flexible. I shall try to read them promptly and return them promptly. If I send you in my grades, perhaps the two of you can combine them with your own. I realize this puts more of the work on you, but I know not what else to do. I do hope we can get the grades in reasonably promptly, and certainly before the end of the quarter, which also means before I return.

Many thanks, and apologies. Best regards too.

Yours,

_________________________

Friedman’s proposed theory exam questions
Summer 1952

M. Friedman

Suggested Questions for Theory Prelim, Summer, 1952

Part I

  1. Define the following terms precisely and indicate briefly the use made of each in economics:
    1. Demand
    2. Supply
    3. Equilibrium
    4. Indifference Curve
    5. Marginal
    6. Rate of Substitution
    7. Marginal value product
    8. Marginal efficiency of capital
    9. Production function
    10. Time preference
    11. Profit
    12. Rent
    13. Run
    14. Net advantages
    15. Variable Costs
  2. (a) “I wouldn’t take it if you paid me”. Draw the consumption indifference curves implied by this statement. (You may find it helpful to suppose first that there is some finite minimum price per unit at which the speaker would take “it”; then approach the limit implied by the quotation.)
    (b) “I’ve reached the point of diminishing returns, so I better quit”. Analyze, indicating under what conditions and for what definition of diminishing returns this is a valid inference from the conditions for a maximum.
  3. (a) Complaints are often heard about the “high” incomes of bootleggers in dry states, or gamblers where gambling is illegal, or smugglers, etc. Are high incomes in such cases evidence of the success or the failure of the laws? Explain your answer.
    (b) A man buys a ticket in a lottery and wins. View this as a business transaction. How much, if any, of his prize is properly regarded as “profit”? Does your answer use the concept of “profit” implicit in the common statement “entrepreneurs seek to maximize profit”? Justify your answer and indicate the difference, if any, between the two concepts.
  4. (a) Outline the theory of joint supply
    (b) What factors determine the elasticity of the derived supply curve of one of a pair of jointly supplied items? Show the direction of influences and prove your statements graphically or otherwise.

*  *  *  *  *  *  *  *  *  *  *  *  *

M. Friedman

Suggested questions for theory prelim, Summer, 1952

Part II

  1. During every hyper-inflation there are always recurrent complaints of a “shortage of money.” How do you explain this phenomenon?
  2. The following quotation is from an article on the illicit gold traffic:
    “Traffic on the Asian gold-smuggling trails has doubled since Korea…Meanwhile savings which could be productively invested by banks lie idle; paper money is snubbed for gold, depreciates with every rise in the gold price, and becomes a weaker and weaker factor in national economies.” (H.R. Reinhardt, The Reporter, July 22, 1952, p.21).
    Analyze this quotation. Precisely what effect would the willingness of people to hold bank deposits instead of gold have on productivity or productive investment, and through what channels? What of sense and what of nonsense is there in the statements after the semi-colon?
  3. There has been much talk of the so-called “wage-price spiral.” What is generally meant by this term? Give a theoretical analysis of the so-called spiral, indicating under what circumstances you think it could or could not arise.

_________________________

Actual Economic Theory Preliminary Examination Questions
Summer, 1952

Summer, 1952

ECONOMIC THEORY I

Time: 4 hours

Answer all questions.

  1. Define the following terms precisely and indicate briefly the use made of each in economics:
    1. Demand
    2. Supply
    3. Indifference Curve
    4. Rate of Substitution
    5. Marginal value product
    6. Marginal efficiency of capital
    7. Production function
    8. Time preference
  2. (a) Outline the theory of joint supply
    (b) What factors determine the elasticity of the derived supply curve of one of a pair of jointly supplied items? Show the direction of influences and prove your statements graphically or otherwise.
  3. Assume that Crusoe is interested in economizing the use of his resources and that during the period in question there is no change in his knowledge of production techniques. How does capital and interest theory aid in explaining the following observations?

(a) After several years, Crusoe begins to obtain berries by planting and cultivation rather than simply by picking them as he had done previously.
(b) After an additional number of years, he reverts to picking wild berries.

  1. What theories do you offer to explain the following phenomena?

(a) During a prolonged rise in the general level of prices, the price of soft drinks remained at five cents with no change whatsoever in the physical characteristics of the product.
(b) During a prolonged rise in the general level of prices the price of candy bars remained at five cents, at the same time, however, as the size of the bars decreased.

  1. Using diagrams, briefly discuss the long-run cost curve for a competitive industry. Indicate, with diagrams, the response to be expected from (a) an expansion of demand, (b) a decrease of demand, within periods too short for a significant change in the fixed investment.
  2. Briefly state the main changes in the body of accepted price theory at the turn from “classical” to “Austrian” (the subjective-value school), i.e., at the “revolution” of the 1870’s. Similarly describe the transition from Austrian to “New-classical” (Marshallian) doctrine.

*  *  *  *  *  *  *  *  *  *  *  *  *

Summer, 1952

ECONOMIC THEORY II

Time: 2 ½ hours

Answer all questions.

  1. During every hyper-inflation there are recurrent complaints of a “shortage of money.” How do you explain this phenomenon? Compare the situation during acute depression.
  2. A part of the nation’s productive capacity is destroyed, say by a war. Ignoring any possible expectational and distributive effects, how will this affect: (a) the division of the national income between consumption and investment? and (b) the income-velocity of money. How, if at all, does your answer depend on whether wealth is a variable which influences behavior?
  3. There has been much talk of the so-called “wage-price spiral.” What is generally meant by this term? Give a theoretical analysis of the so-called spiral, indicating under what circumstances you think it would or would not arise.

_________________________

Theory Prelim, Summer, 1952, Part I. Grades by M. Friedman

General notes:

  1. I have classified the papers into five groups.

P—clear pass for the Ph.D. (7 papers)
P(?) Questionable pass for Ph.D. (5 papers)
A.M. Pass for a.M./questionable fail for Ph.D. (5 papers)
F(?) Questionable fail for A.M., clear fail for Ph.D. (4 papers)
F Clear fail for both (4 papers)

Should emphasize that as always this is somewhat arbitrary. In particular, difference between two fail classes is particularly small in this batch.

  1. In addition to the above class mark, Igive the ranking by my numerical grades. 1 is the best paper, 2, the next best, etc., to aid in seeing whether any differences among members of the committee reflect differences in absolute or relative grading.
# of candidate. Class grade Rank Remarks
1 AM 16
2 F 24
3 P 6
4 P(?) 8
5 P 5
6 F(?) 21
7 AM 14
8 P(?) 11
9 AM 15
10 P 4
11 P(?) 12
12 F 25
13 P 2 This and 15 distinctly the two best papers
14 F(?) 18
15 P 1 See under 13
16 AM 13
17 AM 17
18 F 23
19 F 22
20 P 7
21 P 3
23 F(?) 19
25 P(?) 10
26 F(?) 20
27 P(?) 9

 

PART II OF THEORY PRELIM

Not one of the three papers submitted on this part seems to me satisfactory. #1 is the best of the three, though not by much, and might deserve a questionable pass. Both of the others seem to me clear failures.

_________________________

 

THE UNIVERSITY OF CHICAGO
Chicago 37, Illinois
Department of Economics

September 8, 1952

Mr. Milton Friedman
Orford
New Hampshire

Dear Milton:

Tolley and I have just gone over our three reports and find them fairly well in agreement. The most serious exception is #7—John J. Klein, whose paper you marked passable for the A.M. only, while both Tolley and I gave him a clear pass. Your rank was 14, as you probably have the record to show. What do you suggest? It will be no great hardship to us to re-read the paper, and we shall do so with the next day or so. Do you want to see it again? Or what can we report?

Another questionable case is Adolph Scott (Colored). Here I am the odd man, as I marked him passable, while you ranked him 23 out of 25, and Tolley ranked him 24. I yield as far as passing him for the Ph.D. is concerned but wondered what you would think about passing him for the A.M. He seems to have squeezed through in International Trade at the A.M. level. This would allow him to get the Master’s degree. I feel that these Negroes are in the same position as the Chinese students only more so in that they compete in a completely different market, and they are never really compared with our “full fledged” Ph.D. graduates. (Besides, between you and me, I have attended 4 or 5 Ph.D. exams this summer and thought very few of them ought to pass but they all did).

On Part II there is also some discrepancy. I had Mints read these papers, and he and I agree that #2, Mrs. Mullady, was passable. But you and Tolley both wrote failure and as she failed “flat” on Part I and has also failed a second time in another field, it looks as though that disposes of her case. This leaves S. Smidt who has your vote, a questionable pass, Tolley’s a clear pass, and Mints and I though a very very [sic] dubious pass. But Smidt passes Part I with colors flying. I am perfectly willing and in fact disposed to yield on him and pass him as I don’t feel competent to grade these Part II papers anyway.

Cordially,

(Dictated but not read)
Frank H. Knight

Source: Hoover Institution Archives. Milton Friedman Papers. Box 76. Folder 2 “University of Chicago ‘Economic Theory’”.

_________________________

Identities of eight examinees given passing grades
by Milton Friedman by rank

First place

Seymour Smidt. University of Chicago Ph.D. (1954). Dissertation: “Efficient Management for Government Wheat Stocks”.

Second place

Conrad Jan (Coen) Oort. University of Chicago A.M. (1954). Doctor of Economics, University of Leiden (1958).

Professor economics, U. Utrecht, The Netherlands, 1960-1971; professor economics, University of Michigan, Ann Arbor, 1956-1957; treasurer-general, Treasury, The Hague, The Netherlands, 1971-1977; managing director, Algemene Bank Nederland Bank (now Algemene Bank Nederland-AMRO), Amsterdam, The Netherlands, 1977-1989; non-executive director various companies, The Netherlands, since 1989; professor economics, Maastricht, The Netherlands, since 1986. Chairman KLM, Amstelveen, Netherlands, 1992, Robeco Group, Rotterdam, Netherlands, 1989. Vice chairman Aegon Insurance, The Hague, 1990.
Source: Prabook webpage for Conrad Jan Oort.

Third place

Gary S. Becker. University of Chicago Ph.D. (1953). Dissertation: “The Economics of Racial Discrimination”.
The Sveriges Riksbank Prize in Economic Sciences in Memory of Alfred Nobel 1992.

Fourth place

Lionel John Lerner. [son of Abba P. Lerner and Alice Sendak]. University of Chicago A.B. (1950) and A.M. (1952). Johns Hopkins University Ph.D. (1955). Dissertation: “Theories of Imperialist Exploitation.”
Source: Johns Hopkins University, Sheridan Libraries, Special Collections. Commencement Program 1955, p. 19.

Fifth place

Edward J. Kilberg. Hofstra University B.A. (1949). Duke University A.M. (1952). University of Chicago A.M. (1957).
Apparently Kilberg was never awarded a Ph.D. in economics by the University of Chicago for his dissertation “Commercial bank holdings of cash and liquid items”. Most likely reason is that he died in the crash of a Northeast airliner at Nantucket Airport on August 15, 1958. Kilberg left a research job at the Mutual Life Insurance Company in 1957 to go to the NBER where he worked as assistant to Arthur F. Burns for the book Prosperity Without Inflation (1958).

Sixth place

Hugh Roy Elliott. In the list of economics Ph.D. dissertations kept by the department of economics at the University of Chicago we find “Hugh R. Elliott. Dissertation: Savings Deposits as Money (Summer 1964)” which seems rather late in the game. But then we see: AER Sept. 1957, p. 838 “Hugy [sic] R. Elliott, B.A. Harvard 1950; M.A. Chicago 1952.” Thesis in preparation at Chicago “Savings deposits as money”.

Seventh place

Irwin Ira Baskind. I have found the following item “Baskind, Irwin. Postwar Monetary Policy in Belgium (Ph.D., Chicago)” from U.S. State Department, Bureau of Intelligence and Research. External Research. A List of Studies Currently in Progress, Western Europe, ER list no. 5.14 (April 1960), p. 9. Note: Baskind’s name does not appear in the list of economics Ph.D.’s kept by the Chicago department of economics.

Eighth Place

Paul Gabriel Keat. Baruch School of the City University of New York B.B.A. (1949). Washington University A.M. (1950). University of Chicago A.M. (1952, 1956). University of Chicago Ph.D. (1959). Dissertation: “Changes in Occupational Wage Structure 1900-1956”.

Keat, Paul G. PhD 88, passed away on April 2, 2014.Born in Prague, Czechoslovakia May 2, 1925. A WWII vet who served in Ardennes, Normandy and Rhineland. Decorated with the European African Middle Eastern Services Medal, Good Conduct Medal and WWII Victory Medal. Discharged 1946. Graduated 1959 from the University of Chicago with an M.A. and PhD in economics. Student of his cherished professor, Dr. Milton Friedman. Earned B.B.A. in accounting from Baruch School of the City University of New York and M.A. from Washington University. Paul’s work with IBM was extensive in both the United States and in the European headquarters based in Paris. He taught both finance and economics at the graduate level in numerous universities including Syracuse University, Washington University, the City University of New York, Iona College and the Lubin Graduate School of Business at Pace University. In 2013 he co-authored and published the seventh edition of his textbook “Managerial Economics”.
Source: Arizona Republic, Phoenix. April 13, p. F9.

Images: The economic theory prelim examiners, Friedman, Knight, and Tolley. From the University of Chicago Photographic Archive.

Categories
Chicago Exam Questions

Chicago. Graduate Preliminary Examination, Money and Banking, 1967

 

This copy of the 1967 Money and Banking prelim exam comes from Milton Friedman’s papers and has Milton Friedman’s name noted. So we may strongly presume that Friedman was in fact on the Money and Banking prelim committee as he was on the Income, Employment, and Price Level prelim committee that year.

______________________

Previous posts with University of Chicago preliminary examinations for Ph.D. and A.M.  degrees:

Preliminary Exam (Economic Theory I) 1955

Preliminary Exam (Money and Banking) 1956

Preliminary Exam (Economic Theory) 1957

Preliminary Exam (Money and Banking) 1959

Preliminary Exam (Economic Theory, Old Rules) 1960

Preliminary Exam (Price Theory) 1964

Preliminary Exam (Income, Employment and Price Level) 1967

Preliminary Exam (Price Theory) 1969

Preliminary Exam (Macroeconomics) 1969

Preliminary Exam (Money and Banking) 1969

Preliminary Exam (International Trade) 1970

Preliminary Exam (Price Theory) 1975

Preliminary Exam (Industrial Organization) 1977

Preliminary Exam (History of Economic Thought) 1989

___________________________

[Handwritten note, top of page: “Mr Friedman”]

MONEY AND BANKING
Preliminary Examination for the Ph.D. and A.M. Degrees
Summer, 1967

WRITE THE FOLLOWING INFORMATION ON YOUR EXAMINATION PAPER

—Your code number and NOT your name
—Name of examination
—Date of examination

Results of the examination will be sent to you by letter.

ANSWER ALL QUESTIONS—ALL QUESTIONS HAVE EQUAL WEIGHT

1. a) “The fallacy in the quantity theory of money is that it allows for the circulation of money but not the circulation of goods. A correct theory would have a velocity of circulation of goods to parallel the velocity of circulation of money.” Discuss.

b) According to one writer, one of the “fundamental laws of economics” is that “the inflation rate is approximately equal to the interest rate when averaged over several decades.”
(Andre Gleyzal, “Theory of Money in a Free Economic System.” Discuss (and do not dismiss out of hand).

2. a) What is the “Phillips Curve”?

b) Give the theoretical analysis on which it rests. Do you regard it as valid? If so, defend it; if not, why not?

c) What is its relation to the notion of a “trade-off” between unemployment and inflation?

d) What is your understanding of the present state of the empirical evidence on the Phillips curve?

3. a) Expand the standard analysis of the IS-LM (or EEL) curves to include foreign trade and the balance of payments when all economies are operating with fixed exchange rates under a pure gold standard.

b) Would this analysis be any different under

i) fixed exchange rates with national currency standards?
ii) floating exchange rates?

Why, or why not?

4. a) A once and for all change in the money supply is expected to affect only the price level and not any real economic magnitudes. Yet some economic theorists who accept the neutrality of money in this sense argue that a sudden decrease (say) in the money supply will cause unemployment. How do you reconcile these two positions?

b) Assume that a country is operating on a classical gold standard. It has a central bank but the bank does not engage in open market operations. It confines its policy to setting an interest rate (discount rate) at which it lends freely. Let important gold discoveries be made in that country such that, at the prevailing price of gold, the rate of gold production increases. Does the neutrality of money still hold true in the long run? Will the increased rate of gold production affect only the price level and not the level of real income in the given country?

5. Most empirical studies of the demand for money that use time series data take the real stock of money as the dependent variable and take measures of real income or wealth and of the interest rate as explanatory variables. However, most monetary theorists treat the nominal stock of money as exogeneous. This appears inconsistent with the empirical work. Can you describe a sensible economic model to defend the choice made by the empirical investigators? Assume it is your purpose to predict the increase in the demand for real money balances resulting from an increase in real income. For simplicity, assume that current real measured income is the relevant income variable. Do not discuss the econometric theory of identification, etc. Focus your attention on the economic hypotheses in terms of the price level, the nominal money stock, interest rates, and nominal income. Would it be better to treat real money balances as an explanatory instead of as a dependent variable in estimating the demand for money?

6. Comment on the following proposition:

In the portfolios of banks, private loans and government bonds are alternatives. The smaller the quantity of loans that banks make (i.e., the tighter the supply of bank credit), the greater must be the quantity of government bonds the banks are holding in their portfolios. But the total supply of government bonds is fixed, and so this implies that the tighter is bank credit, the smaller the supply of government bonds available to the non-bank public to hold in their portfolios. But the smaller the quantity of government bonds available to the non-bank public, the greater the quantity of other assets they will hold. In other words, the tighter is bank credit, the greater the supply of private credit from non-bank holders of wealth, and the portfolio behavior of banks is largely irrelevant in determining the total supply of private credit.

Source:  Hoover Institution Archives. Papers of Milton Friedman. Box 77, Folder 8 “University of Chicago Econ. 331”.

Image Source:  “Money Talks” from the cover of Puck, Vol LX, No. 1541 (September 12, 1906). Library of Congress Prints and Photographs Division Washington, D.C.  “William Randolph Hearst sitting with two large, animated, money bags resting on his lap, with arms and legs, and showing two large coins as heads; on the floor next to Hearst is a box labeled ‘WRH Ventriloquist’.”

 

 

Categories
Chicago Exam Questions

Chicago. Preliminary Exam for PhD, Theory of Income, Employment and Price Level, 1967

 

The following preliminary examination for the economics Ph.D. at the University of Chicago comes from Milton Friedman’s papers at the Hoover Institution Archives. Friedman’s own answers for the 20 true-false questions as well as equations for one question and diagrams for another are included below, following the exam.

______________________

Previous posts with University of Chicago preliminary examinations for Ph.D. and A.M.  degrees:

Preliminary Exam (Economic Theory I) 1955

Preliminary Exam (Money and Banking) 1956

Preliminary Exam (Economic Theory) 1957

Preliminary Exam (Money and Banking) 1959

Preliminary Exam (Economic Theory, Old Rules) 1960

Preliminary Exam (Price Theory) 1964

Preliminary Exam (Price Theory) 1969

Preliminary Exam (Macroeconomics) 1969

Preliminary Exam (Money and Banking) 1969

Preliminary Exam (International Trade) 1970

Preliminary Exam (Price Theory) 1975

Preliminary Exam (Industrial Organization) 1977

Preliminary Exam (History of Economic Thought) 1989

______________________

[Handwritten note on top of first page: “Mr. Friedman (grade sheet attached)”]

CORE EXAMINATION
Theory of Income, Employment and Price Level
Summer, 1967

Preliminary Examination for the Ph.D.

WRITE THE FOLLOWING INFORMATION ON YOUR EXAMINATION PAPER:

—Your Code Number and NOT your name
—Name of Examination
—Date of Examination

Results of the examination will be sent to you by letter.

Answer all questions. Time: 3 hours.

 

Part I. Indicate whether each of the following statements is True (T) or False (F) and state briefly your reason. (One hour).  [2 points each]

  1. ____ Free reserves are the difference between total reserves and required reserves.
  2. ____ Member banks may count both currency in vault and deposits at their Federal Reserve Bank as satisfying reserve requirements.
  3. ____ All banks in the U.S. that are members of the Federal Reserve System are required to be members of the Federal Deposit Insurance Corporations, but the reverse is not true.
  4. ____ The Federal Funds rate is the rate at which member banks may borrow from the Federal Reserve System.

5-8: A depositor in a commercial bank transfers funds from a demand deposit to a time deposit at that bank.

  1. ____ The bank’s total reserves are thereby increased.
  2. ____ The bank’s excess reserves are thereby increased.
  3. ____ The amount of currency plus demand deposits that can be outstanding in the System is increased.
  4. ____ The amount of currency plus demand deposits plus commercial bank time deposits that can be outstanding in the System is increased.

 

  1. ____If income velocity of circulation of money is not affected by an increase in real income per capita, then the income elasticity of demand for real balances is zero.
  2. ____ A rise in interest rates can be expected to raise the income velocity of circulation of money.
  3. ____ The real balance effect is absent if all money is “inside” money.
  4. ____ In order for a real balance effect to exist, wealth must be one of the variables entering the consumption function.
  5. ____ The real interest rate can be obtained from the nominal interest rate by dividing by a price index.
  6. ____ The more rapidly the quantity of money grows, the lower will be the quantity of real money balances.
  7. ____ The higher the rate of interest, the lower will be the Keynesian multiplier.
  8. ____ A tariff reduction involves a shift in the IS (or EE) curve associating a lower real income with each interest rate.
  9. ____ A substitution of taxes on property for taxes on earnings (to yield the same revenue at the same national income) will tend to lower national income.

18-20: In the simple income-expenditure model with rigid prices:

  1. ____ A constant positive rate of growth of the quantity of money implies a constant interest rate.
  2. ____ A constant rate of government deficit spending with a fixed stock of money implies a constant interest rate.
  3. ____ A rising stock of capital is inconsistent with a constant interest rate.

 

Part II: Each of the following statements is true. Prove it. (1/2 hour).

  1. The slope of the LM (or LL) curve is flatter, the more elastic the demand for money with respect to the interest rate and the less elastic with respect to income.
  2. Monetary velocity can be expected to be uncorrelated with the level of prices but to be sensitive to the rate of change of prices.
  3. Treasury policy of substituting long term obligations for short-term obligations in the federal debt outstanding will produce deflationary pressure on the economy if and only if the expectations hypothesis about the term structure of interest rates is false or incomplete.
  4. For a given quantity of money, an increase in the government deficit will produce inflationary pressure on the economy, if and only if the elasticity of demand for real money balances with respect to the rate of interest is less than zero.
  5. The usual balanced budget multiplier is unity if and only if liquidity preference is either absolute or depends on income excluding government expenditures.

 

Part III. Consider the following two proposed fiscal policies: (1/2 hour).

(a) Balance continuously the high-employment budget.
(b) Keep tax rates constant.

In considering (a), assume that it can be followed (i.e., that it is possible with at most a brief lag to change taxes in response to changes in government expenditures so that, at high employment, the proceeds of all taxes would equal the amount of expenditure at that level of employment.) Assume also all other conditions, including monetary policy, the same for (a) and (b).

Aside from the effect on the average level of income, which policy do you believe would produce greater stability of income? Justify your answer as rigorously as you can.

 

Part IV. Analyze the likely short and long run effects on interest rates, prices, employment and income velocity of an increase in the rate of monetary expansion from, say, a non-inflationary full employment rate to a higher rate. (1/2 hour).

 

Part V. In a closed economy the central bank can determine the nominal quantity of money, while the public determines its real value, whereas in an open economy the nominal quantity of money is determined by the balance of payments.
Discuss the validity of this statement under alternative assumptions of fixed and flexible exchange rates. (1/2 hour).

*  *  *  *  *  *  *  *  *  *  *  *  *  *

Milton Friedman’s answers and notes

  1. False. = Excess Reserves, Free Reserves = Excess Reserves less Borrowing.
  2. True.
  3. True.
  4. False.
  5. False.
  6. True.
  7. False.
  8. True.
  9. False. Not zero, unity.
  10. True.
  11. True.
  12. True.
  13. False.
  14. True.
  15. True.
  16. True.
  17. Uncertain. Lower W/Y therefore raises savings[?]. [ six words illegible]
  18. False.
  19. True.
  20. False.

Part II.

  1. \begin{array}{l}{{M}^{D}}=f\left( i,y \right)\\{{M}^{S}}=h\left( i \right)\\f\left( i,y \right)=h\left( i \right)\\\frac{dy}{di}=\frac{-\frac{\partial f}{\partial i}+\frac{\partial h}{\partial i}}{\frac{\partial f}{\partial y}}\end{array}

Part V.

 

Source:  Hoover Institution Archives. Papers of Milton Friedman. Box 77, Folder “University of Chicago Econ. 331”.

Image Source:  Element from Social Science Research Building. University of Chicago Photographic Archive, apf2-07449r, Special Collections Research Center, University of Chicago Library.

Categories
Chicago Economics Programs Economists

Chicago. Henry Simons’ Hayek project proposal, 1945

 

Henry C. Simons composed a dozen page, double-spaced, memo that he circulated in draft form to Hayek and the Chancellor of the University of Chicago, Robert M. Hutchins in May 1945. He was afraid that socialists and Keynesians (i.e. the Cowles Commission) were getting the upper-hand and that “traditional-liberal” economists like himself were becoming an endangered species. Not trusting university governing structures, Simons hoped to established an Institute of Political Economy that would dock onto the university but remain an independent beacon of traditional-liberal economics. 

I presumed the unnamed angel in all this was the William Volker Fund, but David Levy thinks the Earhart Foundation would have been a more likely addressee, given the list of people named by Simons. I find it curious that Simons never explicitly mentions a target foundation for his proposal though he had no reservations about including a long list of names of the economists he expected to support the work of his proposed Institute of Political Economy.

Hutchins wrote back to Simons in early September 1945, “I understand from the angel that Hayek has submitted another program, which has no relation to economics.” Simons’ proposal can be considered to have been an elevator pitch for a Chicago-based pre-Mont-Pèlerin Society.

Pro-tip.

According to the University of Chicago Archive’s Guide to the Henry C. Simons Papers, 1925-1972, Box 8, Folder 9 contains Simons’ file regarding his “Institute of Political Economy” proposal. The material for this post all come from Office of the President. Hutchins Administration Records. Box 73, Folder “Economics Dept., 1943-1945”.

______________________
Some of the Backstory

Henry C. Simons Urges his Department Chair (Simeon E. Leland) to Recruit Milton Friedman

August 20, 1945

Henry Simons’ grand strategy was to seamlessly replace the triad Lange-Knight-Mints with his own dream team of Friedman-Stigler-Hart. He feared that outsiders to the department might be tempted to appoint some convex combination of New Dealer Rexford Tugwell and trust-bustin’ George W. Stocking Sr., economists of the institutional persuasion who were swimming on the edges of the mainstream of the time.

______________________

Cover memo from Henry Simons to Robert M. Hutchins

THE UNIVERSITY OF CHICAGO

Date: May 19, 1945

[To:] Robert M. Hutchins

[From:] Henry Simons[,] Department [of] Economics

In re Hayek project

I enclose copies of two memoranda sent to Professor Hayek and of the covering letter.

Hayek asked Friedrich Lutz, Aaron Director, and me to send him suggestions and, when possible, to discuss the matter with one another. Other copies of the enclosures have been sent to Lutz, Director, and a few local people.

When you find time to look at this stuff, you might first read the letter and Memorandum II. The other item (Memorandum I) is long, discursive, and suitable, at best, only for very restricted circulation.

[signed]
Henry Simons

______________________

Henry Simons letter to Friedrich Hayek
[Carbon copy]

 

May 18, 1945

Professor Friedrich Hayek
London School of Economics
The Hostel, Peterhouse
Cambridge, England

Dear Professor Hayek:

I have been struggling to formulate a worthy and promising project that might attract endowment funds. Enclosed find two memoranda which are the poor results of my efforts. Memorandum II is mainly just a condensation of I—and is perhaps better suited for strangers.

I have departed very far from the kind of project we discussed here. I cannot muster or sustain much enthusiasm for any short-term project, or for any project which aims merely at another book or series of tracts. So much good money and professional effort has been wasted on such enterprises. My guess is that one should be less diffident about proposing what one really wants—that one might get both more (and “better”) money and fare better results by projecting something which the active participants might undertake and pursue with conviction and enthusiasm. Honesty is probably the best policy, even when seeking endowment funds.

I have contrived a project largely for what one might call ulterior purposes: (1) to get Aaron Director back here and into a kind of work for which he has, as you know, real enthusiasm and superlative talents; (2) to effect an arrangement regarding visiting professors which I have long espoused. Moreover, I have deliberately formulated the kind of project for which this University would be the natural location and for which Aaron would be a natural choice as head. But I doubt if such ulterior purposes condemn the scheme; on the contrary, the best procedure probably is that of making new schemes to do old things that one has long regarded as desirable. Indeed, the new device, as regards the stream of visitors, has very special merits, for it permits a continuity in the contribution of the visitors which could hardly be achieved otherwise.

I am sorry to have organized Lutz out of the picture—and hope he might be “organized in” again from time to time or permanently. He is probably the best choice for your kind of project; but Aaron seems a better choice for mine, if only by the nature of his own preferences and interests—although Lutz, in turn, would be a better choice for my project if it were located at Princeton.

My scheme may have little or no appeal to the particular donor. I’ve gotten too intrigued with formulating a project to give attention to its saleability to any individual.

We’re still sad about having seen so little of you and about having failed to keep you on for the Summer.

Cordially,

Henry C. Simons

HCS:w
Encl. 2 [Note: only memorandum 1 is to be found in the Hutchins file]

*  *  *  *  *  *  *  *  *  *  *  *  *  *

Memorandum I on a proposed
INSTITUTE OF POLITICAL ECONOMY

It may clarify all that I have to say here if I start with confession of my personal interests and selfish purposes.

A distinctive feature of “Chicago economics,” as represented recently by Knight and Viner, is its traditional-liberal political philosophy—its emphasis on the virtues of dispersion of economic power (free markets) and of political decentralization (real federalism for large nations and for supra-national organization). With the scattering of the “Austrians” and the vastly changed complexion of economics at Cambridge and Harvard, this intellectual tradition (of Smith, Ricardo, Mill, Menger, Wieser, Sidgwick, Marshall, Pigou, Clark, Taussig, Fisher, and Fetter, and of Locke, Hume, Bentham, de Tocqueville, von Humboldt, Acton and Dicey) is now almost unrepresented among the great universities, save for Chicago; and it may not long be well represented at Chicago. It has still many firm adherents, to be sure; but its competent representatives are widely dispersed and isolated from one another, in academic departments or governmental bureaus where they are largely denied opportunity for cooperation with like-minded scholars, or for recruiting and training their successors.

There should, I submit, be at least one university in United States where this political-intellectual tradition is substantially and confidently represented—and represented not merely by individual professors but also by a small group really functioning as a social-intellectual group. This objective presents difficulties, to be sure. Universities will seek to maintain balanced representation of major schools of thought (if not every fashionable novelty), in economics as in other departments; a group of traditional liberals large enough to function effectively might either dominate unduly any single economics department or require, for adequate representation of other “schools,” a department of excessive size. Moreover, traditional labels, individualists in political ideology, tend also to be lone -wolves and excessively individualist in their social-intellectual activities. More than other economists, they must, for real group activity, be selected with regard for their individual propensities for working with one another; if not inordinately friendly and congenial as persons, they are likely to go their separate ways, instead of cooperating, even if propinquity invites a more fruitful community activity.

Consequently, I see much merit in planning for such a group—for such a small social organization of traditional-liberal economists—without total reliance on departmental or university policy and with some loosely or informally affiliated “center” or “institute.” A few traditional-liberal professors might then function both as members of university departments, representing a suitable variety of schools or ideologies and not overlarge, and also as members of a different group centering around the small “institute” or “center” and organized deliberately in terms of a political philosophy or ideology.

Such an institute (Institute for Political Economy) should have a permanent head (Mr. Aaron Director). It should offer services, especially stenographic and mimeographing, for its local group. As its main function, it should, normally in cooperation with the university and department(s), arrange and partly finance extended visits of the best economists and political philosophers of its “school” from all over the world, one or two at a time. It might arrange local lectures or seminar talks by such economists when they happen to be passing through the city. It might sponsor a small local discussion club for faculty, advanced students, and selected outsiders. It might offer a few special fellowships for advanced study—for traditional-liberal economists (teachers, bureaucrats, journalists) as we now offer them for agricultural economists. It might help finance the writing and research of a few cooperating economists not visitors here. Above all, however, it should facilitate the group activity of the interested local professors and maintain a steady flow of competent visitors. From all its activities, a better flow of publications, both scholarly and semi-popular, might be anticipated; but this result should be planned by indirection—stimulated or facilitated rather than required under contracts with participants.

The permanent head of the Institute should be a broadly competent economist, with a major interest in a political philosophy and 19th century English political economic thought. He should be young enough to do creative work and yet mature enough to assure against his stepping out of character as a libertarian. He should be an essentially intellectual person, not a promoter, not politically ambitious or “on the make,” not “the administrator type,” not prominently identified with other organizations or public activity, and not adept at salesmanship or public relations. Indeed, the Institute should have no organized “public relations” at all, should cultivate obscurity, and, while promoting some popular writing, should seek primarily to make its influence felt in the best professional and academic circles, and merely by improving the quality of the writing (and teaching) of individuals. It should not ordinarily engage in publication or seek to identify itself in connection with the publications of its members or participants. Its head should be simply one scholar among scholars, seeking to hold together a group of individuals characterized by common political-economic persuasions, and to help them to help one another—by free interchange of ideas, mutual criticism of preliminary manuscripts, etc.

An important function of the Institute, indeed, should be that of providing typing, mimeographing, and mailing services for affiliated economists. It might facilitate organized discussion (1) of what people intend to write about, (2) of what they have prepared as tentative drafts, and (3) of what they are about ready to publish. Such discussion, besides stimulating writing, should greatly improve its quality, enabling an individual, before publishing, to thresh out disagreements with competent colleagues or, at least, to recognize what their disagreements or dissents are.

The most obvious merit of the scheme, for the University, lives in the plan of bringing in, for extended visits, the best available libertarian economists from other institutions and other countries. Such visitors might mainly or largely be younger men considered more or less eligible for regular appointment to the University faculty. In many cases, the University might be able to “look over” such men without the usual awkwardness of that process—to have them around for six or twelve months without any implied commitment to retain or even to “consider” them for permanent appointment. I should hope that the Institute would, in effect, deeply influence appointments to the faculty, merely by bringing excellent persons whom everyone, knowing them by their visit, would recognize as desirable appointees. It might also improve appointments by itself making this community more attractive to the best candidates.

The closest cooperation between the Institute and the University in the selection of visitors should be maintained. For distinguished visitors nominated by institute, the University might occasionally bear all, and often half, of the cost. For prospective appointees, the University might occasionally use the Institute as a dummy, thus getting a look at the candidate with a minimal [sic] of involvement and without risk of building up expectations that might be unpleasantly disappointed. Normally, it might be hoped that visitors would nominally divide their time between the Institute and the university, each bearing part of the cost.

I naturally would choose Chicago as a location for such an institute, and the University of Chicago as the institution with which to associate it. More substantial reasons than my personal predilections, however, could be offered for this choice. “Chicago economics” still has some distinctively traditional-liberal connotations and some prestige. Here, more than elsewhere, the project would be that of sustaining or keeping alive something not yet lost or submerged—and something which here, too, will shortly be lost unless special measures are taken.

However, I am somewhat open-minded about the location—and should myself be more than ready to go elsewhere, even at financial sacrifice, in order to participate in the kind of intellectual community in question. Likewise, I suspect that many able people might be attracted, at moderate stipends, to any good university where such a prospect was reasonably assured.

And I will concede that the outlook at Chicago, if better than elsewhere, is not very promising. Our Divisional dean has no appreciation of economic liberalism and a distinct hostility toward it, and the same is true of most persons in the other social science departments. Among higher administrative offices, there is at best only indifference, or provisional toleration, toward such political economy. A few members of the Law School and School of Business are interested or sympathetic, as are other individual faculty members here and there. In the department, moreover, we are becoming a small minority. Since I came to the University (1927), only one economist has been appointed who could be classified as really a traditional liberal (he, at an age when cure might still be anticipated); and one (the only fellow I ever found eminently useful as a colleague) was fired simply because of his uncompromising, competent profession of that political-economic philosophy. Meantime, many appointments have been made to the divisional economics staff; and a large staff, overwhelmingly hostile to economic liberalism, has been built up for the College courses in social science. Then, too, we acquired the Cowles Commission and its staff—whose influence the proposed Institute might partially neutralize or offset. Finally, there are our new agricultural economists who, while sympathetic, are real libertarians only avocationally.

Within the large department, there are now Knight, Mints, Viner, myself, and Lewis (in order of age). Knight will soon reach retirement age; Mints is not far behind; and Lewis, long frequently on leave, may well be attracted elsewhere. Moreover, Knight and Viner, while the best of libertarians, can hardly be called members of our group. Knight is increasingly preoccupied with the philosophy and philosophers, not to mention historians, theologians, anthropologists, et al., and is not deeply interested in concrete problems of economic policy. And Viner, while eminently useful to us as Journal editor, seems increasingly to dissociate himself both by interests outside economics and by very special preoccupations in his own writing and research. That leaves Mints and Simons to talk with and to stimulate one another, and to represent libertarian economics on the main teaching front—along with Lewis when he is here. (Viner and Knight teach only quite advanced courses and, even at that level, reach most of the students only in courses which stress technical matters, not political philosophy or political economy.)

On the other hand, our socialist and Keynesian colleagues are friendly and unusually tolerant toward us; and the others are not so much opposed to our political persuasions as simply uninterested—politically neutral or agnostic. It is a group which would be mainly friendly and cooperative with the Institute and its guests; it would doubtless welcome cordially most of the people whom the Institute would propose as visitors, and be happy to use the Institute occasionally for looking over possible appointees. No hostility would be likely to arise if the Institute was properly handled (for its own purposes) and if its resources were moderate.

Let me now formulate more concrete proposals.

(1) The Institute should be projected for roughly a 20-year period.

(2) It should have a permanent head (Aaron Director) with a salary of $7,500—the only person for whom the Institute would hold out permanent, full-time, professional employment.

(3) It should occupy a suite of three or four rooms at 1313 East 60th Street—or, like the Cowles Commission, on the campus—one for the director, one for a secretary-stenographer (or two?) and one for its visiting economists.

(4) It should plan to have one visiting economist (or political scientist, if libertarian ones can be found) on the ground all the time (save for its vacation periods)—and more than one if and as joint appointments and joint financing with the University are arranged.

(5) Finances permitting, it might grant a few fellowships (of, say, $1,000-$1,500) for the advanced training (or refresher training) of persons teaching economics at other institutions, or of interested practicing bureaucrats and journalists.

(6) It might also occasionally bring in outsiders for specific projects of writing and/or research—or assist them in completing publishing work done elsewhere.

(7) It would be highly desirable to have, in addition to the permanent head, a permanent half-time economic statistician, if arrangements could be made for joint appointment, with some department or school of the University, of a suitable person (e.g., Mr. Milton Friedman).

(8) In addition to one or two stenographer-secretaries, generous budgetary provision should be made for peak-load typing and for mimeographing of the manuscripts of economists affiliated with the Institute.

(9) These tentative proposals contemplate a budget of $20,000-$40,000 per year. A start could be made with less than $20,000, and more than $40,000 could easily be utilized effectively; but I distrust munificent arrangements. The important thing financially is assurance of continuity for a considerable period; but, again, I should urge against initial provision for more than 20-25 years. All this implies endowment of $300,000-$600,000—or assurance that funds of that (initial) present value will be steadily available.

The Institute should be set up, not as part of the University of Chicago but independently, with its own governing body and its own funds. It should be located at Chicago, however, only after reasonable assurance of close and friendly relations with the University; and it should be free to move elsewhere if effective or fruitful cooperation later proves unattainable here. The University might undertake to handle Institute funds; it should extend full use of facilities like the Library to the Institute’s director and its guests; it should offer facilities for lectures and seminars sponsored by the Institute; and it should undertake, when feasible, to make temporary (and perhaps one permanent ) joint appointments, so that guests of the Institute might also commonly serve also as members of the faculty. Close administrative cooperation and consultation should be continuously maintained. Cooperation, however, should be achieved largely through individuals, rather than by formal organizational connections.

The Institute should be designed primarily to promote cooperation and communication among competent economists of a traditional-liberal persuasion. It should aim to make such economists more cohesive and more articulate as a group. Its primary concern should be that of contributing to professional discussion and publication at the highest professional level, not that of popularizing or of propagandizing at a mass level. It may be hoped that such publication of popular or semi-popular books and articles would incidentally come about; and some direct efforts to this end would be appropriate. The Institute should seek to focus attention, not only on general economic-political philosophy, but largely on real, concrete problems and issues of public policy. It should, however, adhere firmly to a long and large view of policy, seeking not to influence immediate political action but to improve the quality of discussion of immediate matters. It should largely ignore considerations of immediate political expediency, seeking by discussion to influence professional opinion and thus perhaps to determine what will much later become politically feasible.

The director might properly occupy himself considerably with projects of non-technical writing on major policy problems. He might occasionally arrange for symposium publications, or for a series of special studies, with subsequent summary publications, for a wide audience. In the main, however, the director should be simply one member like others in an academic-intellectual community, contributing his share of talks and manuscripts to the common pool for mutual stimulation and criticism. Like others, moreover, he should publish mainly as an individual.

There are presumably plenty of agencies for publishing and disseminating good popular books and tracts. The Institute might quietly call attention to such writings of libertarian economists as might appeal to other organization; and it might occasionally subsidize or “undisclosedly enterprise” good publications which fail to find other outlets. In the main, however, it should seek to promote work which, when ready for publication, will readily attract commercial publishers. Its subsidies should be largely confined to unusual manuscripts which promise important contribution to professional discussion but do not promise commercially adequate sales.

The Institute, avoiding publicity, should be frank about its purposes and about its ideological position. Its director, its governing board, and all of its consulting or affiliated economists should be chosen as ardent, confirmed free traders—as anti-collectivists, anti-syndicalists, anti-“Planners”—as advocates of free foreign and free domestic trade, of non-discriminatory commercial policies, of untied, non-governmental foreign lending, of deorganization of functional groups, of deconcentration of economic power, of decentralization in national government, of impairment of national sovereignty (through supra-national organization), of devolution of central government powers (in favor of provisional and local powers); i.e., as advocates of systematic and progressive dispersion of power, nationally and internationally. They should be proponents of rigid economy in the kinds of governmental control or intervention—yet more concerned to minimize the kinds than the aggregate amount, and more concerned about minimizing the amount in large or central governments than in local and provincial bodies. Their central credo, following Acton and de Tocqueville, should be that no large organization can be trusted with, or wisely permitted, much power. They should be zealous proponents of the rule of law, of rules of policy as against legislative nose following, and of minimal delegations of discretionary authority. In a word, they should be confirmed constitutional-federalists in the strict sense.

That such an Institute would serve its proper or original purposes cannot be assured for a long period. It can be reasonably assured for (say) twenty years only by the most careful selection of personnel. One can trust Aaron Director to serve such purposes faithfully and intelligently. One can so trust Friedrick [sic] Hayek, Jacob Viner, Frank Knight, Lloyd Mints, Gregg Lewis, Theodore Yntema, Theodore Schultz, Garfield Cox, Wilber Katz, Quincy Wright, Ronald Crane and, to mention some persons elsewhere, Friedrick [sic] Lutz, Herbert Stein, Leland Bach, George Stigler, Allan Wallis, Howard Ellis, Frank Dunston Graham, Frank A. (and Frank W.) Fetter, Harry G. Brown, Joseph Davis, Karl Brandt, Leo Wolman, William A. Paton, Clare Griffin, I. L. Sharfman, Leverett S. Lyon, Milton Friedman, Arthur F. Burns, Gottfried Haberler, Eugene Rostow, Lionel Robbins, Fredrick Bonham, Henry Clay, R. G. Hawtrey, T. E. Gregory, Arnold Plant, A. J. Baster, Colin Clark, Roland Wilson, Harold A. Innis, Carl S. Shoup, James W. Angell, Thurman Arnold, Harry D. Gideonse, Reginald Arragon, Albert G. Hart, John M. Clark and, among prominent business men, William Clayton, and, among journalists, Walter Lippman, John Davenport, and Sir Walter Layton. Many others might be named, and some of those named above could be fully trusted only as members of an otherwise well-selected company.

Aaron Director is not only the ideal person to head the Institute; he is available and would be willing to undertake the task even at financial sacrifice (which he should not be expected to make). He probably would accept the modest stipend compatible with a properly modest and unobtrusive organization. No serious problem should arise in recruiting an able and reliable governing body or a fairly sizable company of conscientious, interested economist-participants or sponsors.

The Institute, to repeat, should not be designed primarily or explicitly as an agency for preparing tracts or reports. It should not be mainly concerned with formal economic theory; neither should it engage substantially in empirical research. It should focus on central, practical problems of American economic policy and governmental structure. It should afford a center to which economist liberals everywhere may look for intellectual leadership or support. It should seek to influence affairs mainly through influencing professional opinion and by preserving at least one place where some political economists of the future may be thoroughly and competently trained along traditional-liberal lines. Money for such causes is perhaps not hard to get and is very easy to spend wastefully or harmfully. In the project here suggested, I can see little danger of miscarriage and real promise of very good results.

______________________

Memo from Merrill Mead Parvis [?] to Hutchins and Colwell

THE UNIVERSITY OF CHICAGO

Date: June 14, 1945

R.M.H. [Robert M. Hutchins]
E.C.C.  [Ernest Cadman Colwell, President of the University of Chicago from 1945 to 1951]

In re Hayek à la Simons

There is an element of fear in Mr. Simons’ presentation of the true faith in economics. It sounds very familiar to me. It weakens any enthusiasm I may have had for the Hayek project. When it is seriously suggested that the staff for the institute should be drawn from men already so old that there is no risk of any ideas entering their heads, the cause must be in precarious condition indeed. Instead of the title that Mr. Simons suggests, I would suggest “asylum for laissez faire economists.”

In the second place, it seems to me that Mr. Simons takes all the vigor out of the proposal: It should not do serious research; it should not produce books that would influence public opinion; but it should aim at being a small, social, intellectual community, effecting contacts and influencing professional opinion. There is an element of dilettantism in this whole proposal, as I read it, that makes it sound like the laissez faire economists dinner club.

The statement of its relationship to the University seems to me to be a very simple one, not altogether desirable. The institute would be a pressure and propaganda group on the edge of the University entirely outside the University’s control, organized for the purpose of forcing or leading the University to appoint orthodox economists. None of this sounds very good for the University to me.

Yours truly,
[signed]

[Guess: Merrill Mead Parvis (1906-1983), colleague of Ernest Cadman Colwell, Chicago Ph.D. 1944, appointed associate professor of New Testament at Emory. Note that Colwell left Chicago in 1951 to become vice president and dean of faculties at Emory University.]

“Colwell was a New Testament scholar of some note. A graduate of Emory University, he received his PhD from the Divinity School at Chicago in 1930. He served on the faculty of the Divinity School from 1930 to 1951. One of his most remarkable decisions was to veto the appointment of George S. Stigler in 1946 to the faculty of the Department of Economics, on the grounds that Stigler was too empirical. See Ronald Coase, “George J. Stigler,” in Edward Shils, ed., Remembering the University of Chicago: Teachers, Scientists, and Scholars (Chicago, 1991), p. 470.

Source: Ftnt. 359 in John W. Boyer The University of Chicago: A History (2015), p. 571.

______________________

Carbon copy

Follow-up Memo from Hutchins to Simons

June 20, 1945

Dear Henry:

Thank you for the memoranda on the Hayek project. What has happened to this scheme?

Sincerely yours,

ROBERT M. HUTCHINS

Mr. Henry Simons
Department of Economics
Faculty Exchange

______________________

(Late) Reply to Hutchins by Simons

THE UNIVERSITY OF CHICAGO

Date: September 4, 1945

Chancellor R. M. Hutchins
From: Henry C. Simons [,] Economics [Department]

I am not remiss in telling you about the Hayek project, for there still is no further news. I have heard nothing from Hayek since he was here—which suggests either that he didn’t like my memos or that he has been preoccupied, possibly as a consultant on the treatment of Germany. Probably something unexpected has happened, for others have heard nothing from him; he is usually more than polite and “correct” about correspondence.

The memos and their scheme, however, were obviously not well contrived to get money from his particular “angel.” I had hopes that they just might be otherwise useful. Now that Sociology and Political Science are going into economics on their own, some scheme like mine is really needed as a counterpoise—not to mention E.H. Carr!

I’m taking the liberty of enclosing copy of a recent memo. [Not found in this file] Let’s hope it is not too irregular to do so, and that you will not be annoyed by passages which, at worst, were not intended to annoy you. Sending copy to you is an afterthought.

[signed, HCS]

HCS-w

P.S. A letter has just come from Hayek. Copy will go to you when it has been deciphered.

______________________

[Carbon copy]

Hutchins’ Reply to Simons

September 10, 1945

Dear Henry:

I understand from the angel that Hayek has submitted another program, which has no relation to economics.

What is the matter with E. H. Carr? I take few exceptions to your memorandum on Economics. My most important one is the implication that the Department is engaged in a bitter struggle with the administration to secure its just desserts. The administration would like nothing better than to make as many first-class appointments in Economics as the Department can prove are first-class.

The implication that the administration has put on pressure for “less good” appointments will prevent the administration from passing on without comment suggestions which it receives from reputable quarters. The suggestion of Stocking came from Edward H. Levi and was sent to Mr. Leland with no comments except those of Mr. Levi.

There is a kind of particularistic flavor about these suggestions for developments in connection with the Cowles Commission, the Law School, and possibly the School of Business, which imply that these are in the central field, whereas Industrial Relations, Agricultural Economics, Political Science and Planning, and possibly American Economic History are not. Some day I want you to explain to me why some of these areas are central and others are ancillary.

But what I started out to say was that I am glad that you are thinking about and pushing for the development of Economics in the University.

Sincerely yours,
ROBERT M. HUTCHINS

Mr. Henry C. Simons
Social Science 516
Faculty Exchange

______________________

The University of Chicago
Department of Economics

October 6, 1945

Chancellor Robert M. Hutchins
Faculty Exchange

Dear Mr. Hutchins:

Your good letter of September 10th was forwarded to me on vacation; hence the tardiness of this reply.

I share most of your disagreements with me! That memo was written for a small group of immediate colleagues—not hypocritically, I hope, but with “slants” that others might easily misinterpret.

I certainly have not felt that the Department is engaged in a bitter struggle with the administration to secure its just desserts. Neither do I object to the passing along of suggestions from reputable quarters. (Levi’s suggestion, by the way, was not without merit, if interpreted as part of his proposal for a large-scale local project in anti-trust investigation.) I was complaining about departmental policy or practice of making no longer-range proposals for recruitment and replacement—not about suggestions coming down to us but about the dearth of suggestions going up.

The Department, I think, should submit to the administration, not only recommendations for immediate, urgent appointments but also a “waiting list,” subject always to revision, of several men whom we definitely want if and when the administration is prepared to act on them. The administration might then make careful, unhurried outside inquiries; and, when outside suggestions are received, we might discuss and report on the relative merits of particular appointments and invite your inquiry on the same basis. Thus the waiting list or appointment program might be kept more or less continuously under critical discussion.

On that matter of what is central and what is ancillary, I think I have an important point, although I might have trouble stating it clearly or persuasively. The point, moreover, is one on which I would anticipate support from you.

About E. H. Carr, I am too strongly and deeply prejudiced for judicious comment. I have seldom reacted so strongly against a book as against his The Conditions of Peace—which is the only Carr book I have read. Knowing nothing of his work on Dostoevski or Bakunin, however, I would have less reason to oppose the appointment if it were in the proposed Russian Institute than if it were in Political Science and International Relations.

My objections to Carr are largely ideological. The Conditions of Peace is a powerful book, very well written and admirable in many parts and aspects. But it is largely and deeply concerned with economics and commercial policy; and here my criticism involves more than bitter disagreement; for here, I think, the fellow is using his rhetorical, journalistic skill to cover up his own lack of insight and understanding. One should not expect all students of politics to discuss economic problems competently. But one may object to their writing arrogantly, caustically, and demagogically about men, books, and subjects that they do not understand.

This book, I think, is one of the outstanding anti-Liberal documents of its time, not only as regards economic policy, domestic and international but also as regards the rights of small nations and their proper place or role in the good society. Carr personifies, for me, almost everything that is wrong with political thinking at both the extreme Right and the extreme Left.

It is significant, I think, that Carr has earned the most bitter denunciation of two such different people as Hayek and Keynes. (Don’t quote me as regards Keynes, for my information is somewhat privileged in that case and second-hand; but I believe it may easily be confirmed.) At best, Carr is a very hot potato in present-day politics—much too hot for wise University appointment, even if one approved of his views.

I should be more diffident about my own reactions to Carr if those of J. Viner and Q. Wright (and Louiee Wright) were not much the same. Incidentally, what is distinctive about Carr (tough political “realism”) is, I think, already adequately represented here, and competently, by Morgenthau.

I’ll be happy to talk sometime about what is central what is ancillary—or as happy as I can be when trying to talk philosophically,

Sincerely yours,

[signed] Henry Simons
Henry C. Simons

ECS-w

P.S. I hear that Milton Friedman, whom I was proposing for Lange’s place, has been appointed to an associate professorship at Minnesota. My scheme thus requires raiding the Minnesota staff for two men, within a few years. Moreover, it might now be best, under that scheme, to get Stigler first.

Source:  University of Chicago Archives. Office of the President. Hutchins Administration Records. Box 73, Folder “Economics Dept., 1943-1945”.

Image Source:  Henry Calvert Simons portrait at the University of Chicago Photographic Archive, apf1-07613, Special Collections Research Center, University of Chicago Library.

 

Categories
Chicago Exam Questions

Chicago. Economic Theory Prelim Exam, Friedman (chair), 1955

 

The examination committee for the Economic Theory prelim given in the summer of 1955 consisted of Milton Friedman (chair), W. Allen Wallis, and D.G. Johnson. Besides the questions, we have some of the answers that are transcribed from Milton Friedman’s handwritten notes from his copy of the examination questions.

Previous posts with University of Chicago preliminary examinations for Ph.D. and A.M.  degrees:

Preliminary Exam (Money and Banking) 1956

Preliminary Exam (Money and Banking) 1959

Preliminary Exam (Economic Theory, Old Rules) 1960

Preliminary Exam (Price Theory) 1964

Preliminary Exam (Price Theory) 1969

Preliminary Exam (Macroeconomics) 1969

Preliminary Exam (Money and Banking) 1969

Preliminary Exam (International Trade) 1970

Preliminary Exam (Price Theory) 1975

Preliminary Exam (Industrial Organization) 1977

Preliminary Exam (History of Economic Thought) 1989

________________________

ECONOMIC THEORY I
Preliminary Examination for the Ph.D. and A.M. Degrees
Summer Quarter 1955

WRITE YOUR NUMBER AND NOT YOUR NAME ON YOUR EXAMINATION PAPER.

Answer all questions. Time: four hours.

  1. (30 points) Indicate whether each of the following statements is true (T), false (F), or uncertain (U). Give a brief explanation of your answer.
    1. ____ If the income elasticity of demand for a product is greater than unity, the relative price of that product will rise as real per capita incomes increase, i.e., will rise relative to products with income elasticity less than unity.
    2. ____ When a firm is producing in a region of rising marginal cost, that firm is in equilibrium because average costs are increasing also.
    3. ____ The market price of steel and iron scrap fluctuates more than the price of finished steel primarily because the scrap market is competitive while the finished steel market is in the hands of monopolists.
    4. ____If automobile firms overproduce and competition forces down the price of new cars, this harms a car owner who has purchased his car on credit since his mortgaged car has suffered a decline in price.
    5. ____ It is frequently stated that the more disagreeable or dirty a job is the more it will be necessary to pay workers, but this is contradicted by the fact that college professors earn more than foundry workers.
    6. ____ Lowering the support price of wheat in the United States at present would aggravate rather than relieve the problem of surpluses, since farmers would simply produce proportionately more in order to maintain their incomes.
    7. ____ An increase in demand for a commodity increases its price, but an increase in price reduces demand. Increases in demand tend, therefore, to be self-compensating.
    8. ____ Increasing the minimum wage rate to one dollar per hour will have little or no effect outside the South, since most workers now being paid less than one dollar per hour are in the South.
    9. ____ In the absence of factors making for an increase in demand, and other things being equal, a new method will be introduced sooner in a competitive than in a monopolized industry.
    10. ____ Without collective bargaining, the workers’ market disadvantage would enable the owners of other productive agencies to appropriate income that would otherwise go to labor.
    11. With collective bargaining, workers in general can appropriate income from the owners of other agents.
    12. ____ In equilibrium, it is enough to know the marginal factor cost of any one factor and its marginal physical product to know the marginal cost of the product, even though the product is produced by many factors.
    13. ____ The demand for a product at the market price is inelastic. It follows that the product must be produced under conditions of net internal diseconomies.
    14. ____ Under competition, the marginal efficiency of capital is equal to the marginal physical product of a particular kind of capital good times the price of the product.
    15. ____ To assert that the rate at which a consumer is willing to substitute x for y decreases as the quantity of x increases along an indifference curve is equivalent to saying that the indifference curve is concave toward the origin.
  2. (10 points) “East coast gas wars are forcing big producers to chop prices to retailers. With some Manhattan service stations selling gas as low as 15.8¢ per gallon, Socony Mobil, Esso Standard Oil and others have cut wholesale prices up to ½¢ per gallon in most of the seaboard marketing area from Maine to Washington, D.C., the first price reduction in nearly a year” Time, July 25, 1955.
    Explain why this quotation is bad economics.
  3. (10 points) Fair trade is now rapidly disappearing. However, a few firms (Sunbeam, Schaeffer) are actively trying to enforce fair trade pricing.
    • (a) Are these firms just misguided or are there circumstances in which fair trade would help them?
    • (b) If fair trade were generally observed, what would be the effect on return on capital and entrepreneurial effort engaged in retailing?
  4. (15 points) A recent court decree requires a company (The United Shoe Machinery Co.) which heretofore has only leased its machines, for which there are at present no competitors, to offer them for sale at prices which will make it neither more nor less advantageous to buy than to rent the machines. How can such prices be determined, and by what criteria can it be determined whether a given price meets the requirement?
  5. (15 points) Discuss the role of “Euler’s theorem” in distribution theory, and give your own position on the issues.
  6. (20 points)
    1. Define (a) perfect competition, (b) oligopoly, (c) monopoly, (d) monopolistic competition, (e) cartel, (f) monopsony.
    2. State the conditions of maximum return for the individual firm in a form in which they are applicable to all the preceding market conditions. Indicate the special form which these take for each of the preceding market conditions.
    3. Define “length of run” and state is effect on these conditions.

*  * *  *  * *  *  * *  *  * *  *  *

Milton Friedman’s Handwritten Notes for Examination

  1. (30 points)
    1. Uncertain. Depends on conditions of supply
    2. False. (blank)
    3. False. Primarily because supply is more inelastic
    4. True. Applies equally to all car owners, whether mortgaged or not
    5. Uncertain. Must allow for extra costs of becoming college professor
    6. Uncertain. Backward (word illegible) supply curve unlikely for crop like wheat with alternative that can be produced instead
    7. False. Confusion of shift in demand and movement along demand schedule
    8. False. affects complements and substitutes in (letter illegible, possibly “N”)
    9. Uncertain. In competitive industry, only necessary that AC of new be less than AC of old which is equal to MC (word illegible) at margin. In monopoly (word illegible) AC of new must be less than MC of old for (3 words illegible).
    10. False. Under competition, no market disadvantage. But (word illegible) that (4 words illegible) enable workers to get larger total income.
      With collective bargaining, workers in general can appropriate income from the owners of other agents.
    11. Uncertain. Depends on elasticity of demand for labor.
    12. True. (blank)
    13. True. if net internal economies, monopoly, which wouldn’t operate at inelastic demand]
    14. False. (not legible)
    15. True

  1. (10 points) (blank)

 

  1. (10 points)

(a) (comment not legible)
(b) Reduce it

  1. (15 points) (blank)

 

  1. (15 points)

1) Exhaustion of product problem—lh;
2) Proves too much;
3) Condition of equilibrium not result of lh.
(“lh” = “linear homogeneity”?)

  1. (20 points)
    1. Definitions. (6 points)
    2. 11 points

2 points for stating the conditions in form applicable to all the market conditions listed in question 1.

1/MR = MPPa/MFCa= MPPb/MFCb= …. = 1/MC

Special form for conditions for

      1. (2 points, perfect competition) MFCa = pa, MR = px
      2. (1 point, oligopoly) (illegible word) MFCa= pa
      3. (1 point, monopoly) MFCa= pa
      4. (1 point, monopolistic competition) same as c.
      5. (2 points, cartel) MFCa= pa, MR not equal MC
      6. (2 points, monopsony) MR = px
    1. (Definition) 1 point; (Effect) 2 points: MFC = infinity or zero for some factors

Source: Hoover Institution Archives. Papers of Milton Friedman, Box 76, Folder “76.2 University of Chicago Economic Theory”.

Image Source:  Milton Friedman (undated) from University of Chicago Photographic Archive, apf1-06230, Special Collections Research Center, University of Chicago Library.

Categories
Chicago Economists Sociology Statistics

Chicago (1907-08). Economist turned Epidemiologist, Edgar Sydenstricker

The last name “Sydenstricker” is certainly not all-too-common which is probably a reason that it lodged in my memory after I transcribed the 25th anniversary of the University of Chicago’s Department of Political Economy. Elgar Sydenstricker was included there in the list of “Fellows of Political Economy”. Nonetheless, I had no record of him ever completing a Ph.D. there (he never did).

With the coming of the Covid-19 pandemic, I thought it might be worth a look to see which economists (if any), were involved in the scientific analysis of the influenza epidemic of 1918-19. The name “Edgar Sydenstricker” was everywhere. And yes, it was the University of Chicago ABD, Edgar Sydenstricker.

I realized there was a significant gap in my rather exclusive focus on Ph.D. academic economists. Someone like Edgar Sydenstricker had an academic economist’s training, but he was not part of the self-perpetuating caste of economics professors.

With the influenza epidemic of 1918-19, Edgar Sydenstricker became a leading statistician in the efforts to advance epidemiology.  Today’s post gives information about his career and publications.

Fun fact: his younger sister was Pearl Sydenstricker Buck (1938 Nobel Prize in literature).

______________________

Best single source about Edgar Sydenstricker
(includes a bibliography)

Kasius R.V., ed. The challenge of facts. Selected public health papers of Edgar Sydenstricker. New York: Prodist, for the Milbank Memorial Fund, 1974.

Wiehl, D.G. Edgar Sydenstricker: a memoir. pp. 1-17.

______________________

Edgar Sydenstricker’s Time-line.
(b. July 15, 1881 in Shanghai; d. Mar 19, 1936 in New York City).

Parents were missionaries from West Virginia, Rev. Dr. Absalom and Caroline Stulting Sydenstricker.

1896. Edgar Sydenstricker came to United States

1900. A.B., Fredericksburg College (Virginia).

1902. M.A. (honors) in sociology and economics at Washington and Lee.

1902-1905. High school principal in Onancock, Virginia

1905.  Editor of the Daily Advance in Lynchburg, Virginia

1907-08. Graduate study at University of Chicago [fellow in political economy]

1908-1915. United States Immigration Commission and Commission on Industrial Relations. Extensive surveys of wages, working conditions, and scales of living of industrial workers, especially in industries with large numbers of foreign born.

1915. Joins United States Public Health Service as first statistician ever. He was hired to assist Dr. B. S. Warren [studied health and economic status of garment workers in New York City, sickness insurance in Europe].

1916-20. Sydenstricker and Joseph Goldberger studied causes of pellagra in the American South.

1917. Elected member of the American Statistical Association.

1918. With Wade Hampton Frost research on statistics of influenza [papers by Sydenstricker, Wade Hampton Frost, Selwyn D. Collins, Rolo H. Britten and others at the Public Health Service giving “a most comprehensive history of influenza from 1910 to 1930”].

1920. Appointed head of Office of Statistical Investigations.

1921. Begins Hagerstown Morbidity Survey [which later became the U.S. National Health ].

1922. Becomes fellow of the American Statistical Association

1923. League of Nations invited him to establish the Epidemiological Service of the Health Organization.

1925. Consultant to Milbank Memorial Fund

1928. Director of research of Milbank Memorial Fund.

1931-34. Represented ASA at Social Science Research Council.

1935. Scientific director of Milbank Memorial Fund

1936, March 19. Died of cerebral hemorrhage.

______________________

The important influenza studies of the Public Health Reports, U.S.

United States Treasury Department and the Public Health Service. Influenza Morbidity and Mortality Studies, 1910-1935. Reprints from the Public Health Reports. Washington: USGPO, 1938.

Influenza-pneumonia mortality in a group of about 95 cities in the United States, 1920-29. By Selwyn D. Collins. Reprint 1355, from Public Health Reports, Vol. 45, No. 8 (February 21, 1930), pp. 361-406.

Influenza and pneumonia mortality in a group of about 95 cities in the United States during four minor epidemics, 1930-35, with a summary for 1920-35. By Selwyn D. Collins and Mary Gover. Reprint 1720, from Public Health Reports, Vol. 50, No. 48 (November 29, 1935), pp. 1668-1689.

Mortality from influenza and pneumonia in 50 large cities of the United States, 1910-29. By Selwyn D. Collins, W. H. Frost, Mary Gover, and Edgar Sydenstricker. Reprint 1415, from Public Health Reports, Vol. 45, No. 39 (September 26, 1930), pp. 2277-2328.

Excess mortality from causes other than influenza and pneumonia during influenza epidemics. By Selwyn D. Collins. Reprint 1553, from Public Health Reports, Vol. 47, No. 46 (November 11, 1932), pp. 2159-2179.

The incidence of influenza among persons of different economic status during the epidemic of 1918. By Edgar Sydenstricker. Reprint 1444, from Public Health Reports, Vol. 46, No. 4 (January 23, 1931), pp. 154-170.

Age and sex incidence of influenza and pneumonia morbidity and mortality in the epidemic of 1928-29 with comparative data for the epidemic of 1918-19. By Selwyn D. Collins. Reprint 1500, from Public Health Reports, Vol. 46, No. 33 (August 14, 1931), pp. 1909-1937.

The influenza epidemic of 1928-29 in 14 surveyed localities in the United States. By Selwyn D. Collins. Reprint 1606, from Public Health Reports, Vol. 49, No. 1 (January 5, 1934), pp. 1-42.

______________________

Other Sydenstricker articles on public health

Edgar Sydenstricker. Existing Agencies for Health Insurance in the United States,” in U.S. Department of Labor, Proceedings of the Conference on Social Insurance, 1916 (Washington, D.C.: U. S. Government Printing Office, 1917), pp. 430-75.

Edgar Sydenstricker. Preliminary Statistics of the Influenza Epidemic, in Epidemic Influenza. Prevalence in the United States. Public Health Reports. Vol. 33, No. 52 ( December 27, 1918), pp. 2305-2321.

Sydenstricker, E., King W.I.A. A method for classifying families according to incomes in studies of disease prevalence. Public Health Reports 1920; 35: 2828-2846.

Sydenstricker, E. Health and Environment. New York: McGraw-Hill, 1933.

Sydenstricker, E. Health and the Depression. Milbank Memorial Fund Q 1934; 12:273-280.

Sydenstricker, E. The incidence of illness in a general population group: General results of a morbidity study from December 1, 1921 through March 31, 1924 in Hagerstown, Md. Public Health Reports. 1925; 40: 279-291.

Milbank Memorial Fund. Program of the Division of Research 1928-1940. (1941)

 

Image Source:  Portrait of Edgar Sydenstricker in Washington and Lee University Yearbook The Calyx, 1902.

Categories
Chicago Economists Gender Iowa

Iowa State. Economics PhD alumna, Alison Comish Thorne, 1939

 

This post is the result of some rummaging in the Iowa State University economics department website, hoping to find material on Albert Gailord Hart for the previous post. While it appears that Hart came and went with hardly a footprint in the Iowa State (web-)sand, I did discover a very nice historical timeline for the Iowa State economics department. Moseying down that timeline, I made the acquaintance of the first economics Ph.D. at Iowa State College, Alison Comish Thorne. Obviously she has meet the membership requirement to be included in our series “Meet an economics Ph.D. alumna”, so I left the Iowa State economics website to search for more about Alison Comish Thorne’s life and career.

Of particular interest for Economics in the Rear-view Mirror is the account of her graduate student experience, especially pp. 24-42 of her autobiography (jstor access required) Leave the Dishes in the Sink (2002). A copy of Alison Comish Thorne’s c.v. is available at a special Utah State University library webpage memorializing her contributions.

_____________________

Selected Early and Late Publications

Thorne, Alison Comish. Capacity to Consume, American Economic Review vol. 26, no. 2 (June, 1936), pp. 292-5.

__________________. Evaluations of Consumption in Modern Thought. Economics Ph.D. thesis, Iowa State College, 1938.

__________________. Evaluations of Consumption in Scale-of-Living Studies, Social Forces vol. 19, no. 4 (May, 1941), 510-518.

__________________. Women mentoring women in economics in the 1930s, in Mary Ann Dimand, Robert W. Dimand, and Evelyn L. Forget, eds. Women of Value: Feminist Essays in the History of Women in Economics (Brookfield: Edward Elgar, 1995), pp. 60-70.

__________________. Leave the Dishes in the Sink—Adventures of an Activist in Conservative Utah. (University Press of Colorado and Utah State University Press, 2002).

_____________________

From the Economics Department Timeline, Iowa State University

Alison Comish Thorne received first PhD

The Doctor of Philosophy in general economics was first offered in 1937; the first PhD was granted to Alison Comish Thorne in 1939. She was the first woman student in the Iowa State economics department to attempt a PhD.

Thorne’s dissertation entitled “Evaluations of Consumption in Modern Thought” was written under Elizabeth Hoyt and Margaret Reid. In the process of working on her PhD, Thorne had an interim year at the University of Chicago, where she studied under Hazel Kyrk.

Thorne’s father, himself an economics PhD, authored a pioneer book in consumer economics and had been a doctoral candidate with Theodore Schultz at the University of Wisconsin.

Alison Comish had married Wynne Thorne in 1937. After earning her PhD at Iowa State, her academic career was delayed not only by the arrival of their five children, but also by anti-nepotism rules at Utah State University, where her husband had become head of agronomy and then director of the ag experiment station and vice president for university research. In addition to being a full-time wife and mother of five, she held state and local elected and appointed positions, served on the Governor’s Committee on the Status of Women, and wrote on employment of women and on poverty. These contributions have been recognized by distinguished service awards from several of these boards and councils and from Utah State University, the American Association of University Women, Business and Professional Women, and Soroptomists. She also received the Governor’s Award for Community Service.

Because administrators’ spouses were not allowed on the faculty, she did not join the USU faculty until 1965, aided in part by the passing of the Civil Rights Act in 1964. She began teaching after an invitation from a professor in the College of Family Life.

After playing a key role in organizing the newly created federal war on poverty programs in Utah, Thorne was invited to teach classes in the USU Sociology Department, as well. Thus, she became a lecturer in sociology, home economics, and consumer education at Utah State University at age 51, 28 years after earning her PhD. She was ineligible for tenure because she insisted on keeping her teaching just under half time in order to give time to her family and to community work.

After joining the staff of USU, she helped initiate the Status of Women Committee and the introductory course in women’s studies, which she taught for more than ten years. She organized and became the first coordinator of Women in International Development (WID).

She continued teaching and doing community work and in 1985, after a university-wide blue-ribbon committee reviewed her credentials, she was promoted to full professor. Because of her age she became “professor emeritus.” With a twinkle in her eye she remarked that she is the only person in the history of Utah State University to leap from lecturer to full professor in one fell swoop.

Of her five children, none became economists, although three became professors. Two of these professors are mothers with husbands in academia, something that would have been impossible in the 1930s.

Source:  Iowa State University, Department of Economics. Compiled by D. Gruca from official university publications and departmental files as well as

I. W. Arthur, “Development of the Field of Economics at Iowa State.”
Nancy Wolff and Jim Hayward, “The Historical Development of the Department of Economics at Iowa State, 1929 to 1985.”
G. Shepherd,“History of Economics at ISU.”

[Also Note: Jim Hayward and Nancy Wolff. The Historical Development of the Department at Iowa State University, 1869-1928.]

_____________________

Women of Caliber, Women of Cache Valley: Alison Comish Thorne

A Woman of Quality

Alison Comish Thorne challenged established perceptions of “womanhood” in order to instigate social change, and admonished other women of her generation to do the same. In a speech she gave in 1949, Alison encouraged women to “let the dishes wait.” She did not want women to lose their sense of personal identity as they fulfilled their roles as wives and mothers. She argued that women should not judge themselves or other women based on the tidiness of their homes. Alison demonstrated for women of Cache Valley that achieving an education and pursuing a career while being a wife and mother could be a reality. She balanced her professional responsibilities with her family duties and received personal fulfilment from both.

Alison was a trailblazer in the world of female higher education. Her pursuit for higher education began at a young age. In 1930, at sixteen years old, Alison attended Brigham Young University. In 1934, she graduated with a Bachelor’s degree in Economics and Education. Then, in 1935, she earned a Master’s degree in Consumption Economics from Iowa State University. In 1938, Alison became the first woman to receive a Ph. D. in Consumption Economics from Iowa State University.

Second Wave Feminism and the Equal Rights Amendment

Ahead of her time, Alison brought second wave feminism to Cache Valley. Along with many other women during the mid-twentieth century, Alison took upon herself the legacy of Alice Paul, an early-twentieth century suffragette and author of the original Equal Rights Amendment (ERA). When first introduced in 1923, the original ERA championed for both men’s and women’s rights, but took into consideration “women’s distinct needs.” The amendment’s objective was to establish men and women as equal under the law and focused on the right of women to compete equally with men in “all aspects of social and economic life.” Alice Paul opposed “protective legislation”—gender based laws written with the intention of “protecting” women from exploitation that, in reality, prevented women from pursuing work in particular professions, limited the number of hours they were allowed to work, and restricted pay rates. Despite Alice Paul’s valiant effort, the amendment did not pass.[1]

“Equality does not mean sameness.”

The ERA Alison promoted offered an updated version of Paul’s original amendment. Alison’s version of the ERA raised the issues of access to higher education, participation in the draft, and sexual discrimination within the Social Security program. In a draft for a pamphlet designed to promote the ERA in Utah to ratify, Alison explained, “The Amendment supports the constitutional equality for women and the extension of legal rights, privileges, and responsibilities regardless of sex.”

Similar to the movement in the Progressive Era, the ERA movement of the 1970s faced fierce competition from conservative groups such as “Humanitarians Opposed to Degrading Our Girls” (HOTDOG), “International Women’s Year” (IWY), and “Women for Maintaining the Difference between the Sexes and Against the Equal Rights Amendment.” In a pamphlet for the 1977 IWY Convention, the association announced that it opposed the ERA because the amendment “would provide undefined limits of governmental power over the lives of its citizens.” The IWY supported the idea that a government should have limited power over its citizens. The LDS church also aggressively campaigned against the ERA, a stance that divided LDS women. By opposing the ERA, many LDS women “outwardly revealed to each other their internal acceptance of the church’s teaching about proper gender roles.” Those who supported the ERA seemingly questioned church doctrine and ignored the counsel of church leadership.[2] Alison tried diligently to reassure members of the church that their religious rights would not be impinged. Equality did not mean that men and women became “the same.” From Alison’s point of view, the ERA provided women equality under the law, protected “traditional” roles of women, and simultaneously offered women more way to navigate life as established definitions of “womanhood” were being challenged.

[1] Amy E. Butler, Two Paths to Equality: Alice Paul and Ethel M. Smith in the ERA Debate, 1921-1929 (New York: University of New York Press, 2002), 1-2.

[2] Neil J. Young, “’The ERA Is a Moral Issue’: The Mormon Church, LDS Women, and the Defeat of the Equal Rights Movement,” American Quarterly 59, 3 (September 2007): 625; O. Kendall White, Jr., “Mormonism and the Equal Rights Amendment,” Journal of Church and State 31.2 J (1989): 249-268.

Source: Utah State University, University Libraries. Digital Exhibit, Women of Caliber, Women of Cache Valley: Alison Comish Thorne.

_____________________

Biographical Note from Archives

Alison Comish Thorne was born May 9, 1914 in Chicago, Illinois, the daughter of Newel H. and Louise Larson Comish. Her scholarly pursuits began at the age of sixteen when she entered Brigham Young University where she earned her Bachelor’s degree in Economics and Education in 1934. Thorne received a Master’s degree in Consumption Economics at Iowa State University in 1935. She then pursued doctoral studies at the University of Chicago during 1935-36, before receiving her Ph.D. in 1938 from Iowa State University in the field of Consumption Economics. Her mentors, Elizabeth Ellis Hoyt and Margaret G. Reid, worked with Thorne to help her become the first woman to receive a Ph.D. in this field from ISU. Thorne married D. Wynne Thorne on August 3, 1937 in Salt Lake City.

After the completion of her graduate work, Thorne filled various instructor positions at Colorado State University, Iowa State University, and finally Utah State University. At USU she was given the title of lecturer from 1964 through the 1980s by both USU’s Department of Sociology and the Department of Home Economics and Consumer Education. Due to anti-nepotism laws, Thorne was not allowed to secure a faculty position since her husband was already a faculty member. (Wynne Thorne served as USU’s Head of Agronomy, Director of the Agricultural Experiment Station, and the Vice President of University Research.) This setback did not keep Thorne from establishing a solid reputation as a scholar. Thorne played a key role in the founding of the Women’s Studies Program at USU and served as a chair in the Women’s Studies Committee from 1977-1989. In addition, Thorne’s devotion to increasing the opportunity for women can be seen in her involvement in the Women’s Center, the Committee on the Status of Women, as well as the Women and International Development committee.

Moreover, Thorne gave many early feminist speeches, including “Let the Dishes Wait” (1949) and “Leave the Dishes in the Sink” (1973). These speeches encouraged women to focus more on personal hobbies, interests, education, and family rather than maintaining a “perfect” home. As result of her influential work, Thorne has been the recipient of many awards, such as Utah State University’s Distinguished Service Award (1982), Woman of the Year for the Utah Chapter of the American Association of University Women (1967), and Utah Governor’s Award for Volunteer Service (1980). She was also the author of numerous articles and books, including Women in the History of Utah’s Land-grant College (1985), Visible and Invisible Women in Land-grant Colleges (1986), Vision and Rhetoric in Shakespeare: Looking Through Language (2000), Leave the Dishes in the Sink: Adventures of an Activist in Conservative Utah (2002), and Shakespeare’s Romances (2003).

Thorne was active in many organizations during her retirement, such as the Utah State Historical Society, the Utah State Women’s History Association, and the National Women’s Studies Association. Thorne died in 2005 in Logan, Utah.

Source: See Archives West: Utah State University, Papers of Alison Comish Thorne, 1925-2003.

Image Source: Detail from the cover of Alison Comish Thorne’s Leave the Dishes in the Sink (2002).

Categories
Chicago Economists Princeton

Chicago to Princeton. Jacob Viner’s Resignation Letter, 1946

 

 

Jacob Viner was 53 years old when he decided to leave the University of Chicago for Princeton. The letter transcribed below presents the personal and professional reasons for his resignation. Viner’s letter reveals the soft note of a late mid-life crisis motivating the move — no need for a new wife and fancy sports car, but rather the scholar sought retreat to an ivory-tower library, comfort from the families of his adult children and a significantly shorter commute to points East. 

___________________

Resignation Letter of Jacob Viner

COPY SENT TO ALL MEMBERS OF THE COMMITTEE ON INSTRUCTION AND RESEARCH.
ORIGINAL TO MISS STROMWALL

 

THE UNIVERSITY OF CHICAGO
Chicago 37, Illinois
Department of Economics

January 24, 1946

Chancellor Robert M. Hutchins
Office of the Central Administration
Faculty Exchange

Dear Mr. Hutchins:

I am sorry to have to tell you that I am hereby resigning from the University of Chicago as of the end of this academic year to take a professorship at Princeton University. As I am to be at the London School of Economics from April to July, I am leaving the University in March, on leave without pay for the spring quarter.

The Princeton offer first came to me over a year ago, and I definitely declined it twice. It has been renewed and I now have definitely accepted it. For many reasons the offer had great attractions for me from the start, and I am sure that it was loyalty to Chicago which led me at first to decline it — as was true also in the case of Yale. I have been associated with the University of Chicago for 30 years, and the University has always treated me, both before and since your regime, with the utmost generosity. I have never in the thirty years had any cause to feel a personal grievance against you, or any other administrative officer, or any colleague. I am not leaving because of any dissatisfaction with the way the University has treated me, or with my colleagues, with you, or with any of your aides. I did disapprove at times of some aspects of what I thought to be your policy for the University. I still do on some points. But I had my full day in court; and my disapproval of some of your ideas — which it is not impossible that I misunderstood — never carried with it any lessening of my respect for yourself, or any ill-will toward you, and was always accompanied by enthusiastic approval of other of your educational ideas. I have been very happy about the way the new Council of the University has been operating, and if there were any criticism I would venture to offer of your behavior as its presiding officer, it would be that you have been too self-effacing and have not given as much leadership as the Council would welcome and would benefit from. I assure you that when I leave it will be with the friendliest feeling toward you and with strong affection for the University which I so cherish and which has dealt so kindly with me.

Why then am I leaving? A variety of factors have played their part. Most important, probably, is that our children, although born here, never took root in Chicago, that our daughter is already living in New York, and that our son, when he gets out of the Coast Guard, is sure of only one thing, that he will not live in Chicago. They have been pressing us very hard to move East, and what with their four years at eastern colleges, Arthur’s military service, and Ellen’s removal to New York, we have not been able to have the family together except for very short intervals over a period of some seven years. While we don’t expect them to live in Princeton, we do hope that we will at least have them frequently for week-end and vacation guests.

As for myself, as I get older, the physical aspects of the routines of teaching and of committees, of inadequate secretarial help, of a library physically difficult to use, and of frequent inescapable trips East, have grown progressively more burdensome to me. I know that as time goes on, these matters will become still more important to me. In the past ten years I have done what looks to me like a great deal of research, but most of it was done while I was on leave of absence from the University and I have put little of it into print. Even when I was in Washington, the secretarial and library privileges I had enabled me to do nearly as much of my personal research in my spare time as I could do in all my time while at the University.

It is not that the University imposed unfair burdens on me. Some of my heaviest chores were self-imposed. But at Chicago I did not want special privileges which equally-deserving colleagues did not have, and some of the things I wanted badly I would for this reason not have accepted at Chicago if they had been offered to me. At Princeton I come now, without obligations going beyond my contract, and with no reason for not accepting all the facilities voluntarily offered me. If after a while I should acquire Princeton loyalties which consume time and energy, I will at least have had a long spell during which I had been free to realize some of my ambitions as a scholar. I want to be able to get some of my accumulated research into shape for print, and if I don’t succeed, it will not be Princeton’s fault, provided it delivers on all the things I have been promised. These things are not financial: my salary will be the same at Princeton as here, and was not an issue. It’s all a matter of easy access to a library easy to use, light teaching load, no journal to edit, no burdensome eastern trips for committees, government advice, work on eastern library collections, etc., a secretary, a research assistant, a good office layout, and so forth. As I could not ask you to build a new library building for me, or to give me facilities that my colleagues could not get, or to transfer the University to the East, I thought it would be easier for all concerned if I did not let anyone here  know about my negotiations.

I may be wrong in deciding that I will be happier and a more productive scholar at Princeton. No matter how well things go there, I know that I will always have a warm feeling for Chicago and a regret that I did not prove to be one of those rare creatures: a professor who passed all his working life at a single institution of which he was not a graduate. I will always be ready upon request to render any service to the University within my power, and I will carry out scrupulously my obligations to Chicago students who are working on theses under my direction or have any other claims upon me.

I hope it will be satisfactory to you if President Dodds announces my appointment on February 1st. I think it only fair to resign at once as a member of the Council and also as a member of the Social Science Research Committee where I would be participating in the making of decisions without staying to take the blame should there be any. I will continue on my other committees at your pleasure and the pleasure of their Chairmen until I leave.

I have never written an academic letter of resignation before. If there is any further action I need to take, I trust you will let me know. I am sending a copy of this letter to Dean Redfield, who has also deserved better of me.

Very sincerely yours,
/a/ Jacob Viner

JF-w

Source: University of Chicago Library, Special Collections Research Center. Office of the President, Hutchins Administration Records 1892-1951. Box 73, Folder “Economics Department, 1946-1950”.

___________________

Noted Economist Is Appointed To Princeton Staff

Princeton, Feb. 27. The appointment of Professor Jacob Viner, of the University of Chicago, internationally known as one of America’s most distinguished economists and editor of the “Journal of Political Economy,” to the faculty of Princeton University was announced today by President Harold W. Dodds. He will join the Princeton faculty next September.

“The addition of Professor Viner to the Department of Economics and Social institutions will contribute to the attractiveness of the university as a center of under graduate and post-graduate studies. President Dodds said. “He is distinguished as a professor and as an economist in the fields of international finance and trade. The program of our International Finance Section will benefit, but his usefulness to the university will not be limited to the activities of the section. He is a scholar whose Interests embrace the whole field of economics.”

Professor Viner, a native of Montreal, received his B.A. degree from McGill University in 1914; his M.A. from Harvard University in 1915, his Ph.D. from Harvard in 1922 and an LL.D. degree from Lawrence College in 1941.

He was a professor of economics at the University of Chicago, 1916-17; special expert with the U.S. Tariff Commission, 1917-19, and the U.S. Shipping Board during part of 1918. He returned to the University of Chicago in 1919 as an assistant professor of economics, was appointed associate professor in 1923 and professor in 1925. Since 1940 he has been Morton Hull Distinguished Service Professor at that institution.

Meanwhile, he has served variously as visiting professor at the Institute Universitaire de Hautes Etudes Internationales, Geneva. Switzerland, 1930-31 and 1933-34 visiting professor at Yale University, 1942-43; special assistant to the Secretary of the Treasury, dur[ing] part of 1934; consulting expert to the U.S. Treasury, 1935-39; special assistant to the Secretary of the Treasury during parts of 1939 and 1942; and consultant U.S. Department of State since 1943.

Professor Viner is a fellow of the American Academy of Arts and Sciences and the American Philosophical Society. He is a member and former president of the American Economical [sic] Association and the author of: “Dumping a Problem in International Trade”; “Canada’s Balance of International Indebtedness”; “Studies in the Theory of International Trade”; and numerous articles on economic subjects.

Source: The Morning Call (Paterson, N.J.), February 28, 1946. Page 7.

Image Source: Jacob Viner (left); Theodore W. Schultz (right).  University of Chicago Photographic Archive, apf1-07483, Special Collections Research Center, University of Chicago Library.